Disaster, Emergency and Trauma Nursing, Critical care, Terrorism, casualty

Réussis tes devoirs et examens dès maintenant avec Quizwiz!

Yesterday, the client was weaned from the mechanical ventilator and an intravenous infusion of lorazepam. The client has been alert and oriented for 24 hours but is now experiencing confusion. The nurse now evaluates new-onset confusion by assessing the client's sense of place and time, difficulty focusing, short-term memory loss, and increasing lethargy. The nurse suspects which condition in this client? 1 Amnesia 2. Delirium 3. Dementia 4 Psychosis

2

The intensive care nurse is caring for a client who has just been extubated. Which interventions are appropriate at this time? Select all that apply. 1. Administer prescribed oral narcotics for throat pain 2. Administer warmed, humidified oxygen via facemask 3. Give the client ice chips to moisten the mouth 4. Provide mouth care with oral sponges 5. Start the client on incentive spirometer

2,4,5

Which client event would be considered an adverse event and would require completion of an incident/event/irregular occurrence/variance report? Select all that apply. 1. Administered 9:00 AM medication at 9:30 AM 2. Developed worsening cellulitis after missing antibiotics for 1 day 3. Has a seizure and a history of epilepsy 4. Slides off the edge of the bed and ends up sitting on the floor 5. Waits 4 hours to be transported for STAT diagnostic CT scan

2,4,5

You are an Emergency Department nurse who has to care for three victims of anthrax. The first victim inhaled the toxin, the second victim ingested it, and the third victim suffered a skin infection. Which client should be cared for first? a) The one who ingested the toxin b) The one who inhaled the toxin c) The one with the skin infection d) Any convenient order

B. the one who inhaled the toxin

When attending a client with a head and neck trauma following a vehicular accident, the nurse's initial action is to? A Do oral and nasal suctioning. B. Provide oxygen therapy. C Initiate intravenous access. D Immobilize the cervical area.

D Immobilize the cervical area. Question 20 Explanation: Clients with suspected or possible cervical spine injury must have their neck immobilized until formal assessment occurs. Options A, B, and C: Suctioning, oxygen therapy, and intravenous access are also done after the cervical spine is immobilize.

4

24. The off-duty nurse hears on the television of a bioterrorism act in the community. Which action should the nurse take first? 1. Immediately report to the hospital emergency room. 2. Call the American Red Cross to find out where to go. 3. Pack a bag and prepare to stay at the hospital. 4. Follow the nurse's hospital policy for responding.

A client has been exposed to cyanide. Which of the following would the nurse expect to assist in administering? Select all that apply. a) Amyl nitrate b) Sodium thiosulfate c) Atropine d) Dimercaprol e) Sodium nitrite

A. amyl nitrate B. sodium thiosulfate E. sodium nitrite

A patient with hypotension and temperature elevation after doing yard work on a hot day is treated in the ED. After the nurse has completed discharge teaching, which statement by the patient indicates that the teaching has been effective?

"I should have sports drinks when exercising outside in hot weather." rationale: Electrolyte solutions such as sports drinks help replace fluid and electrolytes lost when exercising in hot weather. Salt tablets are not recommended because of the risks of gastric irritation and hypernatremia. Antipyretic medications are not effective in lowering body temperature elevations caused by excessive exposure to heat. A patient who is confused is likely to have more severe hyperthermia and will be unable to remember to take appropriate action.

When assessing a patient admitted to the emergency department (ED) with a broken arm and facial bruises, the nurse notes multiple additional bruises in various stages of healing. Which statement or question by the nurse is most appropriate?

"Is someone at home hurting you?" rationale: The nurse's initial response should be to further assess the patient's situation. Telling the patient not to return home may be an option once further assessment is done. The patient, not the nurse, is responsible for reporting the abuse. A social worker may be appropriate once further assessment is completed.

What helps the emergency department (ED) nurse develop priority-setting skills in a nonthreatening environment? Select all that apply. 1 Human/client simulation 2 Simulation software 3 Hands-on clinical experience in the ED 4 Case study discussions 5 Shadowing a physician

-Human/client simulation -Simulation software -Case study discussions Case study discussions and the use of human/client simulation and simulation software help the ED nurse to acquire priority-setting skills in a nonthreatening environment. The ED is not considered a nonthreatening environment, so gaining hands-on clinical experience in the ED can be ruled out. Shadowing a physician will not help the nurse develop priority-setting skills because the physician's role is not within the nurse's scope of practice.

A client's wife is panic-stricken at the thought of withdrawing all life support from her husband, who is dying from end-stage chronic obstructive pulmonary disease and sepsis. She asks the nurse what he will experience when mechanical ventilation is stopped. Which statement made by the nurse is most appropriate at this time? 1. "The healthcare provider will prescribe a continuous intravenous infusion of morphine to make him more comfortable." 2. "To maintain blood flow to his heart and lungs, we will continue norepinephrine, the vasopressor, but discontinue all other medications." 3. "To prevent aspiration, we will discontinue his feeding tube and begin total parenteral nutrition to meet his nutritional needs." 4. "We will continue basic care, such as monitoring his vital signs, giving nutrition, and monitoring laboratory tests."

1

During assessment of a client who had major abdominal surgery a week ago, the nurse notes that the incision has dehisced and evisceration has occurred. The nurse stays with the client while another staff member gets sterile gauze and saline. How should the nurse position the client while waiting to cover the wound? 1. Low Fowler's position with knees bent 2. Prone to prevent further evisceration 3. Side-lying lateral position 4. Supine with head of the bed flat

1

The client is brought to the emergency department after falling off a roof and landing on his back. A T1 spinal fracture is diagnosed. The client's blood pressure is 74/40 mm Hg, pulse is 50/min, and skin is pink and dry. What nursing action is a priority? 1. Administer IV normal saline 2. Determine if urinary occult blood is present 3. Perform a neurological assessment 4. Verify that there is no stool impaction

1

The nurse is caring for a client who is 1 day postoperative extensive abdominal surgery for ovarian cancer. The client is receiving IV Ringer's lactate at 100 mL/hr and continual epidural morphine for pain control. The Foley catheter urine output has decreased to <20 mLhr over the past 2 hours. The postoperative hematocrit is 36% (0.36), and the hemoglobin is 12 gIdL (120 g/L). Which action should the nurse carry out first? 1. Assess vital signs 2. Increase the IV rate to 125 ml/hr 3. Notify the health care provider 4. Perform a bladder scan

1

The nurse notes a change in the condition of a client in septic shock with an infected leg ulcer and positive blood cultures for methicillin-resistant Staphylococcus aureus. Which assessment finding is most important for the nurse to report to the health care provider? 1. Cold and clammy skin 2. Oxygen saturation of 92% 3. Sinus tachycardia of 118/min 4. Urine output of 0.5 mL/kg/hr

1

The nurse performs admission assessments on 4 clients. Which client assessment information is most concerning and needs priority care? 1. 17-year-old with suspected meningococcal meningitis who has a fever of 103 F (39.4 C), headache with photophobia, and stiff neck 2. 36-year-old who is an IV drug user with cellulitis of the arm, a fever of 103.2 F (39.6 C), and foul-smelling drainage from self-injection sites 3. 45-year-old with diabetes mellitus and osteomyelitis of the foot who has a fever of 100.9 F (38.3 C) and a serum glucose of 295 mg/dL (16.4 mmoll) 4. 76-year-old with chronic bronchitis who has a fever of 101 F (38.3 C) and a productive cough of thick green mucus

1

Following an earthquake, patients are triaged by emergency medical personnel and are transported to the hospital. Which of these patients will the nurse need to assess first?

A patient with a red tag rationale: The red tag indicates a patient with a life-threatening injury requiring rapid treatment. The other tags indicate patients with less urgent injuries or those who are likely to die.

Between 1980 and 2001, terrorists have attacked the United States and its territories via bombings. What type of injury generally results from these attacks, which account for nearly 70% of all terrorist attacks? a) Penetrating, blunt trauma b) Radiation burns c) Radiation poisoning d) None of the options are correct

A. penetrating, blunt trauma

After losing her home to a hurricane several years ago, the client says, "I get very nervous during a thunderstorm and want to hide under the bed." What is the nurse's best response? a. "But it is just a thunderstorm. You would have warning if a hurricane was approaching." b. "I understand. That is normal and is nothing to worry about." c. "That is post-traumatic stress disorder. You might want to see a counselor." d. "What do you do when you feel this way?"

D (Check to see whether the thought is causing maladaptive behavior, and then assess whether it is normal or abnormal.)

critical care 18 hotspot http://author.udutu.com/myudutu/preview/previewcourse.aspx?CourseID=137491&ScreenID=4335829

c

The provider is planning to discharge a client home. The nurse suspects domestic violence as the cause of injury, although the client denies this. What is the best course of action for the nurse to take? 1 Call the police. 2 Consult with Social Services. 3 Discharge the client as instructed. 4 Instruct the client to go to a safe place.

Consult with Social Services. If discharge home is not deemed safe, the client may be admitted to the hospital until resources can be organized to provide a safe environment. Social workers or case managers are consulted to investigate resource needs and plan accordingly. Calling the police is not an appropriate response. Letting the client go home could place the client in danger. The client may not have a safe place to go.

A client brought to the trauma center dies following a suicide attempt. What action does the nurse take to prepare the body for family viewing? Incorrect1 Remove all IV and indwelling tubes. 2 Invite the family to assist in cleaning the body. 3 Hand over objects from the client's pocket to the family. 4 Cover the body and only expose the face.

Cover the body and only expose the face. The nurse should cover the body and only expose the face of the client in a dimly lit room for viewing by the family. The client's death following a suicide attempt may require forensic investigation or a medical examiner's case. The nurse does not remove IV and indwelling tubes, ask the family to assist in cleaning the client's body, or hand over objects from the client's pocket to the family as these actions could potentially damage evidence.

During the primary assessment of a trauma victim, the nurse determines that the patient is breathing and has an unobstructed airway. Which action should the nurse take next?

Observe the patient's respiratory effort. rationale: Even with a patent airway, patients can have other problems that compromise ventilation, so the next action is to assess the patient's breathing. The other actions also are part of the initial survey but assessment of breathing should be done immediately after assessing for airway patency.

These four patients arrive in the emergency department after a motor vehicle crash. In which order should they be assessed?

c) A 22-year-old with multiple fractures of the face and jaw a) A 72-year-old with palpitations and chest pain b) A 45-year-old complaining of 6/10 abdominal pain d) A 30-year-old with a misaligned right leg with intact pulses rationale: The highest priority is to assess the 22-year-old patient for airway obstruction, which is the most life-threatening injury. The 72-year-old patient may have chest pain from cardiac ischemia and should be assessed and have diagnostic testing for this pain. The 45-year-old patient may have abdominal trauma or bleeding and should be seen next to assess circulatory status. The 30-year-old appears to have a possible fracture of the right leg and should be seen soon, but this patient has the least life-threatening injury.

A patient who is unconscious after a fall from a ladder is transported to the emergency department by family members. During the primary survey of the patient, the nurse should _____________

obtain a Glasgow Coma Scale score. rationale: The Glasgow Coma Scale is included when assessing for disability during the primary survey. The other information is part of the secondary survey.

A triage nurse in a busy emergency department assesses a patient who complains of 6/10 abdominal pain and states, "I had a temperature of 104.6º F (40.3º C) at home." The nurse's first action should be to ______________

assess the patient's current vital signs. rationale: The patient's pain and statement about an elevated temperature indicate that the nurse should obtain vital signs before deciding how rapidly the patient should be seen by the health care provider. A urinalysis may be needed, but vital signs will provide the nurse with the data needed to determine this. The health care provider will not order a medication before assessing the patient.

A patient who has experienced blunt abdominal trauma during a car accident is complaining of increasing abdominal pain. The nurse will plan to teach the patient about the purpose of ______________

ultrasonography. rationale: For patients who are at risk for intraabdominal bleeding, focused abdominal ultrasonography is the preferred method to assess for intraperitoneal bleeding. An MRI would not be used. Peritoneal lavage is an alternative, but it is more invasive. An NG tube would not be helpful in diagnosis of intraabdominal bleeding.

When planning the response to the potential use of smallpox as an agent of terrorism, the emergency department (ED) nurse-manager will plan to obtain sufficient quantities of ______________

vaccine. rationale: Smallpox infection can be prevented or ameliorated by the administration of vaccine given rapidly after exposure. The other interventions would be helpful for other agents of terrorism but not for smallpox.

When preparing to rewarm a patient with hypothermia, the nurse will plan to _____________

attach a cardiac monitor. rationale: Rewarming can produce dysrhythmias, so the patient should be monitored and treated if necessary. Urinary catheterization and endotracheal intubation are not needed for rewarming. Sympathomimetic drugs tend to stimulate the heart and increase the risk for fatal dysrhythmias such as ventricular fibrillation.

critical care 24 drag and drop http://author.udutu.com/myudutu/preview/previewcourse.aspx?CourseID=137491&ScreenID=4304818

c

critical care 40 exhibit http://author.udutu.com/myudutu/preview/previewcourse.aspx?CourseID=137491&ScreenID=4337067

c

critical care 46 drag and drop http://author.udutu.com/myudutu/preview/previewcourse.aspx?CourseID=137491&ScreenID=4335298

c

critical care 53 exhibit http://author.udutu.com/myudutu/preview/previewcourse.aspx?CourseID=137491&ScreenID=4335834

c

Based on the progress note documentation, which priority intervention does the nurse anticipate? Click on the exhibit button for additional information. Exhibit Progress notes 2000 Client admitted to CCU #4, reporting vise-like chest pain and shortness of breath. Pulmonary artery (PA) catheter inserted by the health care provider via right internal jugular vein without difficulty. Central venous pressure (CVP) 18 mm Hg, pulmonary artery wedge pressure (PAWP) 25 mm Hg and coarse crackles auscultated bilaterally. ______________, RN 1. 0.9% sodium chloride, 500 mL intravenous bolus 2. Furosemide, 40 mg intravenous push 3. Metoprolol, 5 mg intravenous push 4. Vancomycin, 1 g intravenously every 12 hours

2

The nurse is caring for a client who had a near-drowning accident in cold weather. Which assessment finding indicates the most severe injury? 1. Decreased body temperature 2. Toes pointed straight down 3. Weak and thready pulse 4. Wheezing on auscultation

2

When caring for a client with a left radial artery catheter, which assessment data obtained by the nurse indicates the need to take immediate action? 1. Capillary refill of less than 3 seconds 2. Left hand cooler than right 3. Mean arterial pressure of 65 mm Hg 4. Pressure bag at 300 mm Hg

2

Which client incident would be classified as an adverse event that requires an incident/event/irregular occurrence/variance report? Select all that apply. 1. Client admitted with white blood cell count of 28,000 mm3 (28.0 × 109/L) and dies from sepsis 2. Client receives 1 mg morphine instead of prescribed 0.5 mg morphine 3. Client refuses pneumonia vaccination and contracts pneumonia 4. Nurse did not report client's new hemoglobin result of 6 g/dL (60 g/L) to oncoming nurse 5. Provider was not notified of client's positive blood culture results

2,4,5

You are assessing a patient who has sustained a cat bite to the left hand. The cat is up-to-date immunizations. The date of the patient's last tetanus shot is unknown. Which of the following is the priority nursing diagnosis? A Risk for Impaired Mobility related to potential tendon damage. B Risk for Infection related to organisms specific to cat bites. C Ineffective Health Maintenance related to immunization status. D Impaired Skin Integrity related to puncture wounds.

B Risk for Infection related to organisms specific to cat bites. Cat's mouths contain a virulent organism, Pasteurella multocida, that can lead to septic arthritis or bacteremia. Options A and D: There is also a risk for tendon damage due to deep puncture wounds. These wounds are usually not sutured. Option C: A tetanus shot can be given before discharge.

In an emergency situation, how is the incident of an overturned school bus categorized? a. Not a mass casualty b. May be a mass casualty c. Mass casualty d. Act of terrorism

B (A mass casualty event overwhelms local medical capabilities. It may require the collaboration of multiple agencies and health care facilities to handle the crisis. Depending on the community, available resources, and the quantity and severity of those injured, this may be a mass casualty.)

Which essential item is added to a personal readiness supplies "go bag"? a. Fruits and vegetables b. Potable water c. Television d. Laptop computer

B (The go bag should contain 1 gallon of potable water per person per day.)

After successful treatment of clients involved in a mass casualty incident, the incident commander deactivates the emergency response plan. Which of these activities is most important for the emergency department (ED) charge nurse to initiate at this time? a. Analyze the ED response to the mass casualty incident. b. Take inventory, and re-stock the ED with supplies and equipment. c. Initiate critical incident stress debriefing (CISD) for staff members. d. Follow up with survivors to determine the need for additional referrals.

B (The priority is re-stock the ED to return to normal operation.)

Three victims of radiation exposure are brought into the Emergency Department. As the nurse caring for these clients, you would expect what substance to be ordered to reduce radiologic organ damage? a) Cyan red b) Potassium iodide c) Medical iodine d) Russian blue

B potassium iodide

When dealing with a radiation leak at your hospital, you know that you must do what to prevent the spread of the contaminants? a) Floors must be scrubbed. b) Air ducts and vents must be sealed. c) Double gloving is not required. d) Waste must be contained within the facility.

B. air ducts and vents must be closed

A nurse is providing care to a client who has been exposed to phosgene vapor. Which nursing diagnosis would the nurse identify as the priority? a) Disturbed sensory preception: visual related to bilateral miosis and visual disturbances b) Impaired gas exchange related to destruction of the pulmonary membrane c) Impaired skin integrity related to vesicant contact with skin d) Decreased cardiac output related to altered aerobic metabolism from agent exposure

B. impaired gas exchange related to destruction of the pulmonary membrane

Anthrax acquired by which of the following methods develops into the most severe? a) Ingestion b) Inhalation c) Skin infection d) Contact with body fluids or contaminated objects

B. inhalation

Exposure to gamma radiation can be decreased by completing which of the following actions? a) Wearing thick clothes b) Providing distance from radiation source c) Lengthening time of exposure d) Providing plastic shielding

B. providing distance from radiation source

You are a nurse caring for clients in the emergency department who have been admitted from the area surrounding a nuclear power plant. There had been a small explosion at the plant and a small amount of radiation had escaped. You know that this is what type of a disaster? a) Natural b) Radiologic c) Chemical d) Explosive

B. radiologic

Prevention is a tool we all can use to decrease our potential for gamma radiation contamination in the event of a nuclear accident or attack. Being aware of ways to limit external contamination empowers individuals to feel safer and to protect themselves and their loved ones. If caught outside during such an event, which of the following is the most effective way to decrease the potential for external gamma radiation contamination by an individual? a) Don't worry, gamma radiation is harmless. b) Remove all garments and shoes before entering a house or public shelter. c) Turn on window fans or air conditioners to blow out contaminated air. d) Cover the mouth and nose with a scarf or handkerchief.

B. remove all garments and shoes before entering a house or public shelter

When preparing for an emergency bioterrorism drill, the nurse instructs the drill volunteers that each biological agent requires specific patient management and medications to combat the virus, bacteria, or toxin. Which of the following statements reflect the patient management of variola virus (smallpox)? a) A vaccination is effective only if administered within 12 to 24 hours of exposure. b) Smallpox spreads rapidly and requires immediate isolation. c) Acyclovir is effective against smallpox. d) Smallpox is spread by inhalation of spores.

B. smallpox spreads rapidly and requires immediate isolation

A client with a gunshot wound is admitted to the emergency department (ED). Which minimum Standard Precaution activity does the nurse require for staff safety? 1 Blood and body fluid precautions 2 Metal detector screening of the client 3 Placement of a security guard 4 Use of a positive air-purifying respirator (PAPR)

Blood and body fluid precautions The ED nurse uses Standard Precautions at all times when there is the potential for contamination by blood or other body fluids. Screening of the client with a metal detector, appointing a security guard, and use of a PAPR, although beneficial, are not minimum Standard Precautions.

The emergency department (ED) triage nurse is assessing four victims of an automobile accident. Which patient has the highest priority for treatment?

A patient with a sucking chest wound rationale: Most immediate deaths from trauma occur because of problems with ventilation, so the patient with a sucking chest wound should be treated first. Face and head fractures can obstruct the airway, but the patient with facial injuries has lacerations only. The other two patients also need rapid intervention but do not have airway or breathing problems.

How does the high school nurse react directly after a random shooting at a high school? a. Actively listens to student b. Assesses his or her own individual feelings c. Encourages students to vent feelings d. Facilitates community cohesion

B (One must be able to support oneself before supporting others.)

The nurse is assessing a client admitted to the trauma center following an accident in which the client fell through the ice on a frozen lake. What action does the nurse take to prevent hypothermia after the client's clothing is cut off? 1 Set the room temperature to 95° F (35° C). 2 Infuse IV solutions at room temperature. 3 Provide blankets and use heat lamps. 4 Infuse blood products at room temperature.

Provide blankets and use heat lamps. The nurse should provide the client with blankets and use heat lamps after removing any wet sheets or clothing to prevent hypothermia from setting in. The room temperature should be set at 75° F to 80° F (24° C to 27° C) and not 95° F. IV solutions and blood products are warmed before infusion to prevent hypothermia.

The wounded victim is unable to walk, has respiratory rate of 12, capillary refill is 8 seconds, and is unresponsive. The wounded victim is assigned what tag color? A. Green B. Red C. Yellow D. Black

The answer is B: Red.

In the intensive care unit, the nurse cares for a client who is being treated for hypotension with a continuous infusion of dopamine. Which assessment finding indicates that the infusion rate may need to be adjusted for the client? 1. Central venous pressure (CVP) is 6 mm Hg 2. Heart rate is 120 beats per minute (bpm) 3. Mean arterial pressure (MAP) is 78 mm Hg 4. Systemic vascular resistance (SVR) is 900 dynes/sec/cm-5

2

3

22. The nurse is caring for a client in the prodromal phase of radiation exposure. Which signs/symptoms should the nurse assess in the client? 1. Anemia, leukopenia, and thrombocytopenia. 2. Sudden fever, chills, and enlarged lymph nodes. 3. Nausea, vomiting, and diarrhea. 4. Flaccid paralysis, diplopia, and dysphagia.

6. Proper hand placement for chest compressions during cardiopulmonary resuscitation (CPR) is essential to reduce the risk of which complication? 1. Gastrointestinal bleeding. 2. Myocardial infarction. 3. Emesis. 4. Rib fracture.

4. Proper hand placement during chest compressions is essential to reduce the risk of rib fractures, which may lead to pneumothorax and other internal injuries. Gastrointestinal bleeding and myocardial infarction are generally not considered complications of CPR. Although the victim may vomit during CPR, this is not associated with poor hand placement, but rather with distention of the stomach.

You are the nurse caring for three clients who have been diagnosed with anthrax. They were exposed after boarding a flight where a white powdery substance was found in one of the restrooms. You know that these clients would be classed as being victims of which of the following? a) A natural disaster b) A biologic disaster c) A chemical disaster d) A radiologic disaster

B. a biologic disaster

The emergency department charge nurse is making client assignments and delegating care after a mass casualty event. Care for which of these clients could be delegated to a nursing assistant? a. A client who has multiple left rib fractures and is complaining of dyspnea b. A client who is complaining of severe left anterior chest pain c. A client who has a femoral fracture with palpable distal pulses d. A client who is unconscious and has massive aortic bleeding from the chest

D (This client is unlikely to survive owing to massive thoracic bleeding and would be "black-tagged" and assigned to a nursing assistant.)

A 15-year-old male client was sent to the emergency unit following a small laceration on the forehead. The client says that he can't move his legs. Upon assessment, respiratory rate of 20, strong pulses, and capillary refill time of less than 2 seconds. Which triage category would this client be assigned to? A Black. B Green. C Red. D Yellow.

D. Yellow. Question 16 Explanation: The client is possibly suffering from a spinal injury but otherwise, has a stable status and can communicate so the appropriate tag is YELLOW.

critical care 16 hotspot http://author.udutu.com/myudutu/preview/previewcourse.aspx?CourseID=137491&ScreenID=4335823

c

critical care 28 exhibit sata http://author.udutu.com/myudutu/preview/previewcourse.aspx?CourseID=137491&ScreenID=4335831

c

A nurse is preparing an in-service education program to a group of nurses who are members of a disaster response team specializing in biologic weapons. Which of the following would the nurse include as the agent of choice when dealing with a mass casualty incident involving anthrax? a) Gentamicin b) Erythromycin c) Ciprofloxacin d) Penicillin

C. ciprofloxacin

In relation to submersion injuries, which task is most appropriate to delegate to an LPN/LVN? A Talk to a community group about water safety issues. B Stabilize the cervical spine for an unconscious drowning victim. C Remove wet clothing and cover the victim with a warm blanket. D Monitor an asymptomatic near-drowning victim.

D Monitor an asymptomatic near-drowning victim. The asymptomatic patient is currently stable but should be observed for delayed pulmonary edema, cerebral edema, or pneumonia. Options A and B: Teaching and care of critical patients are an RN responsibility. Option C: Removing clothing can be delegated to a nursing assistant.

critical care 65 exhibit http://author.udutu.com/myudutu/preview/previewcourse.aspx?CourseID=137491&ScreenID=4335836

c

The oncoming nurse is receiving report on 4 clients. Which should be the priority assessment? 1. Client who had a carotid endarterectomy that day with a blood pressure of 160/88 mm Hg 2. Client who is 1 day post bowel resection with absent bowel sounds 3. Client with a pulse of 109/min who has a history of atrial fibrillation 4. Client with pancreatitis whose total parenteral nutrition is almost finished

1

Upon arrival in the post-anesthesia care unit (PACU), the nurse performs the initial assessment of a client who had surgery under general anesthesia. Which assessment finding prompts the nurse to notify the health care provider (HCP) immediately? 1. Difficult to arouse 2. Muscle stiffness 3. Pinpoint pupils 4. Temperature 96 F (35.6 C)

2

Four clients were involved in a major highway motor vehicle accident. Which client requires priority care? 1. Client with blood pressure of 90/70 mm Hg and deviated trachea 2. Client with concussion who was unconscious for 5 minutes 3. Client with grossly swollen upper thigh and blood pressure of 80/60 mm Hg 4. Client with pain at the thoracic spine and complete paralysis of both legs

1

How does the nurse ensure that discharge instructions are helpful to the client? 1 Assess the client's visual acuity and reading level. 2 Ensure that instructions are no higher than a 10th- grade reading level. 3 Provide instructions in English for clients who speak English as a second language. 4 Provide large print materials for all clients.

Assess the client's visual acuity and reading level. When providing discharge instructions, the nurse should assess the client's visual acuity and reading level to determine the print size suitable for the client, then ensure that discharge instructions are customized to address the client's needs. Instructions must be provided at no higher than a 6th grade reading level. Educational materials are preferably provided in the client's primary language; interpreters may be necessary to customize the information correctly if the material is not available in the client's primary language. Large print materials are provided for older clients and clients with visual deficits.

Which of the following is defined as the potential of an agent to cause injury to the body? a) Persistence b) Toxicity c) Latency d) Volatility

B. toxicity

An alarm has reached your ED regarding a serious MVA between a full tour bus and a school bus; the number of casualties expected is quite high. As you reach the site and begin your assessments, you find many abrasions and lacerations. Which of the following nursing interventions are required to maintain the skin integrity when caring for clients in disaster situations? a) Apply cold water over the wound. b) Administer a prescribed colony-stimulating agent. c) Administer IM antibiotic to prevent wound infection. d) Apply a semi occlusive dressing over wounds.

D. apply a semi occlusive dressing over wounds

Since September, 2001, the United States has had terrorism thrust into its national consciousness. What main strategy do terrorists use to manipulate the politics and policies of a nation? a) Attacking military positions b) Bombing government buildings c) Beheading military personnel d) Frightening and maiming civilians

D. frightening and maiming civilians

An 18-year-old is brought to the emergency department (ED) with multiple lacerations and tissue avulsion of the right hand. When asked about tetanus immunization, the patient denies having any previous vaccinations. The nurse will anticipate administration of ______________

TIG and tetanus-diphtheria toxoid and pertussis vaccine (Tdap). rationale: For an adult with no previous tetanus immunizations, TIG and Tdap are recommended. The other immunizations are not sufficient for this patient.

You're working as a triage nurse during a disaster situation. Based on the triage color code tags placed on each of the wounded, which tag color represents the wounded who have the highest priority of being treated first? A. Green B. Yellow C. Red D. Black

The answer is C: Red. The red tag indicates the patient must be seen first because they have life-threatening injuries, but could survive if treated quickly. The patient is still alive but there is a severe alteration in their breathing, circulation, or mental status that requires immediate medical attention.

The wounded victim is unable to walk, has respiratory rate of 19, capillary refill of one second, and is able to obey your commands. The wounded victim is assigned what tag color? A. Green B. Red C. Yellow D. Black

The answer is C: Yellow.

The wounded victim is unable to walk, respiratory rate is absent and when airway is repositioned breathing is still absent. The wounded victim is assigned what tag color? A. Green B. Red C. Yellow D. Black

The answer is D: Black.

While triaging the wounded from a disaster, you note that one of the wounded is not breathing, radial pulse is absent, capillary refill >2 seconds, and does not respond to your commands. What color tag is assigned? A. Green B. Red C. Yellow D. Black

The answer is D: Black. The black tag is placed on the wounded that are dying or have expired. The injuries are so severe that death is imminent. There is severe alteration or absence of breathing, circulation, and neuro status.

An older client admitted to the emergency department is on a stretcher awaiting bed availability. What intervention does the nurse perform to protect the skin integrity of this client? 1 Provide warm sheets and cover the client with a blanket. 2 Use mobility techniques that reduce shearing force when moving the client. 3 Confirm that side rails are up and wheels are locked on the stretcher. 4 Wash hands frequently or use hand sanitizers when caring for the client.

Use mobility techniques that reduce shearing force when moving the client. The nurse uses mobility techniques that reduce shearing force when moving the client to protect the client's skin integrity. Providing warm sheets and covering the client with a blanket keeps the client warm but does not protect skin integrity. Siderails should be up and wheels locked on the stretcher to prevent the risk for falls in the client who may be disoriented and confused. Frequent handwashing or the use hand sanitizers help to prevent pathogen transmission.

A pipe bomb detonated on a city bus, causing numerous casualties. This would be an example of which type of disaster? a) Unintentional human b) Biologic c) Natural d) Intentional human

A. intentional human

During a disaster, the nurse sees a victim with a green triage tag. The nurse knows that the person has which type of injuries? a) Minor and treatment can be delayed hours to days b) Significant and require medical care, but can wait hours without threat to life or limb c) Extensive and chances of survival are unlikely even with definitive care d) Life-threatening but survivable with minimal intervention

A. minor and treatment can be delayed hours to days

A nurse who is working as part of a disaster response team is performing triage at a mass casualty incident. One of the victims has a sucking chest wound. The nurse would triage this client using which color-coded tag? a) Red b) Black c) Yellow d) Green

A. red

A client in the emergency department is placed on a stretcher. What precautions does the nurse take to prevent further injury to this client? Select all that apply. 1 Keep the rails up on the stretcher. 2 Provide an identification bracelet. 3 Ask a family member to remain with the client. 4 Remind the client to use the call light/bell for assistance. 5 Obtain a thorough client and family history.

-Keep the rails up on the stretcher. -Ask a family member to remain with the client. -Remind the client to use the call light/bell for assistance. When caring for a client on a stretcher in the emergency department, the nurse should keep the rails up on the stretcher, ask a family member to remain with the client if necessary, and remind the client to use the call light or bell for assistance. These measures help to prevent accidents when the client is on the stretcher. An identification bracelet with two unique identifiers is provided to identify the client. A thorough client and family history is obtained to prevent risk for errors and adverse events during client care.

An 80-year-old client is being discharged from the emergency department to home. What action does the nurse take to prevent future emergency visits from this client? Select all that apply. 1 Provide large-print discharge care instructions. 2 Evaluate client prescriptions and over-the-counter medications. 3 Assess the client and caregiver for risk of falls at home. 4 Ensure that the client has an advance directive. 5

-Provide large-print discharge care instructions. -Evaluate client prescriptions and over-the-counter medications. -Assess the client and caregiver for risk of falls at home. The nurse should assess this client and caregiver for risk of falls at home. The client should also be given large-print discharge care instructions for easy readability. The client's prescription and over-the-counter medications are evaluated to determine if the drug regimen is to be continued. An advance directive is important at the time of the client's admission to a hospital or health care facility, not at discharge. The physician would be contacted at the time of the client's admission to the hospital.

What are the responsibilities of the emergency department (ED)? Select all that apply. 1 Survey public health. 2 Offer not more than 25 inpatient beds. 3 Provide around-the-clock emergency care. 4 Provide medical care to uninsured clients. 5 Provide health care to communities far from hospitals.

-Survey public health. -Provide around-the-clock emergency care. -Provide medical care to uninsured clients. The ED is responsible for public health surveillance and emergency disaster preparedness. It provides around-the-clock emergency care. The ED provides medical care to insured and uninsured clients, thus functioning as a safety net for all citizens. The Centers for Medicare and Medicaid Services (CMS) designates rural facilities with not more than 25 inpatient beds as critical access hospitals. These hospitals provide 24/7 emergency care to community residents that are not close to any other hospitals in the region.

A 13-year-old is brought to the emergency department (ED) after being in a motor vehicle collision. The client was struck in the face by the airbag and reports black spots floating in the field of vision. What intervention should the ED nurse complete first? 1. Cover both eyes with patches 2. Make client NPO 3. Notify the health care provider 4. Place client on bed rest

1

A client who is 2 hours post aortic valve replacement is in the intensive care unit (ICU). The low pressure alarm for the client's radial arterial line sounds. Which action should the nurse take first? 1. Check for bleeding at tube connection sites 2. Perform a fast flush of the arterial line system 3. Re-level the transducer to the phlebostatic axis 4. Zero and re-balance the monitor and system

1

A client with a blood pressure (BP) of 250/145 mm Hg is admitted for hypertensive crisis. The health care provider prescribes a continuous IV infusion of nitroprusside sodium. Which of these is the priority goal in initial management of hypertensive crisis? 1. Decrease mean arterial pressure (MAP) by no more than 25% 2. Keep blood pressure at or below 120/80 mm Hg 3. Maintain heart rate (HR) of 60-100/min 4. Maintain urine output of at least 30 mL/hr

1

A client with acute respiratory distress syndrome is receiving positive pressure mechanical ventilation with 15 cm H2O (11 mm Hg) positive end-expiratory pressure (PEEP). The nurse should assess for which complication associated with PEEP? 1. Barotrauma 2. Decreased oxygen saturation 3. Hypertension 4. Oxygen toxicity

1

A client with blunt trauma undergoes an exploratory laparotomy to repair the intraabdominal injury. After 24 hours, the client has a nasogastric tube attached to continual low suction, 2 Hemovac closed-wound suction abdominal drains, and is receiving IV Ringer's lactate and continual epidural morphine. The client now develops hypotension, tachycardia, oliguria, and severe nausea. What is the client's priority nursing diagnosis (ND) at this time? 1. Deficient fluid volume 2. Impaired urinary elimination 3. Nausea 4. Risk for infection

1

A client with hypothermia has just arrived in the emergency department via ambulance. The client is being rewarmed with blankets, and the IV fluids are being changed over to warmed fluids. What additional intervention is a priority? 1. Attaching the cardiac monitor 2. Covering the client's head 3. Drawing blood for electrolytes and glucose 4. Placing an additional large-bore IV catheter

1

33. The nurse notices a pair of nervous-acting individuals entering the emergency department. When reporting suspicious activity, the nurse should include which of the following in the report? Select all that apply. 1. Vehicle/s description. 2. Current location of parties involved. 3. Names and phone numbers of parties involved. 4. Relationship to hospitalized client. 5. Tone of voice of each party involved.

1, 2 . Vehicle/s description. 2. Current location of parties involved. All suspicious individuals or activities should be reported as soon as possible to the secu- rity department. When reporting an incident, nurses/ employees should provide the ollowing: (a) type o incident; (b) persons involved/physical descrip- tion; (c) vehicles involved and description; (d) date and time the incident occurred; (e) location where the incident occurred; () weapons involved; and (g) current location o parties involved. All reports o threats, actual episodes o violence, or suspicious individuals or activities must be investigated.

20. A small airplane crashes in a neighborhood of 10 houses. One of the victims appears to have a cervical spine injury. What should first aid for this victim include? Select all that apply. 1. Establish an airway with the jaw-thrust maneuver. 2. Immobilize the spine. 3. Logroll the victim to a side-lying position. 4. Elevate the feet 6 inches (15.2 cm). 5. Place a cervical collar around the neck

1, 2 1. Establish an airway with the jaw-thrust maneuver. 2. Immobilize the spine. The victim o a neck injury should be immobilized and moved as little as possible. It is also important to ensure an open airway; this can be accomplished with the jaw-thrust maneuver, which does not require tilting the head. The victim should not be rolled to a side-lying position nor have his eet elevated. Both actions can cause additional injury to the spinal cord. Placing a cervical collar causes movement o the spinal column and should not be done as a rst-aid measure.

34. There has been an increase in medication errors and errors in prescribing laboratory studies in the emergency department. The nurse manager is conducting a staff education session on when to use "read-back" procedures. "Read-back" procedures should be performed in which of the following situations? Select all that apply. 1. When a medication prescription or critical laboratory result is received verbally or over the telephone. 2. When any verbal or phone prescription is received. 3. Whenever a written prescription or printed critical test result is received. 4. When the unit secretary takes a phone prescription. 5. When the agency uses computerized health care records.

1, 2 1. When a medication prescription or critical laboratory result is received verbally or over the telephone. 2. When any verbal or phone prescription is received. A goal o client saety is to improve the eectiveness o communication among caregivers. For verbal or telephone prescriptions, or or tele- phone reporting o critical test results, one must veriy the complete prescription or test result by having the individual receiving the inormation record "read-back" the complete prescription or test result. The Unit Secretary is not a licensed health care proessional who has a Scope o Practice or the authority to receive prescriptions or results. The type o charting system used by the health care agency is not a actor in using "read-back" prescriptions.

18. An airplane crash results in mass casualties. The nurse is directing personnel to tag all victims. Which information should be placed on the tag? Select all that apply. 1. Triage priority. 2. Identifying information when possible (such as name, age, and address). 3. Medications and treatments administered. 4. Presence of jewelry. 5. Next of kin.

1, 2, 3 1. Triage priority. 2. Identifying information when possible (such as name, age, and address). 3. Medications and treatments administered. Tracking victims o disasters is important or casualty planning and management. All victims should receive a tag, securely attached, that indicates the triage priority, any available identiying inormation, and what care, i any, has been given along with time and date. Tag inorma- tion should be recorded in a disaster log and used to track victims and inorm amilies. It is not necessary to document the presence o jewelry or next o kin.

27. A number of clients have come to the emergency department after a possible terrorist act of arsenic overexposure. The nurse should assess these clients for which signs or symptoms immediately following the poisoning? Select all that apply. 1. Violent vomiting. 2. Severe diarrhea. 3. Abdominal pain. 4. Sensory neuropathy. 5. Persistent cough.

1, 2, 3 1. Violent vomiting. 2. Severe diarrhea. 3. Abdominal pain. When arsenic overexposure occurs, the signs and symptoms include violent nausea, vomiting, abdominal pain, skin irritation, severe diarrhea, laryngitis, and bronchitis. Dehydration can lead to shock and death. Ater the acute phase, bone marrow depression, encephalopathy, and sensory neuropathy occur.

12. The nurse in the emergency department reports there is a possibility of having had direct contact with blood of a client who is suspected of having HIV/AIDS. The nurse requests that the client have a blood test. Consent for human immunodefiency virus (HIV) testing can only be completed when which of the following circumstances are present? Select all that apply. 1. An emergency medical provider has been exposed to the client's blood or body fuids. 2. Testing is prescribed by a physician under emergency circumstances. 3. Testing is prescribed by a court, based on evidence that the client poses a threat to others. 4. Testing is done on blood collected anonymously in an epidemiologic survey. 5. When a health care provider who is taking care of a client who is suspected of having HIV/AIDS requests a blood test.

1, 2, 3, 4. Upon a physician's written prescription requesting an HIV test for a client, consent for HIV testing must be obtained. Consent exceptions include the following: testing is prescribed by a physician under emergency circumstances, and the test is medically necessary to diagnose or treat the client's condition; testing is prescribed by a court, based on clear and convincing evidence of a serious and present health threat to others posed by an individual; testing is done on blood collected or tested anonymously as part of an epidemiologic survey; or an emergency medical provider has been exposed to the client's blood or body fluids.

15. An explosion at a chemical plant produces flames and smoke. More than 20 persons have burn injuries. Which victims should be transported to a burn center? Select all that apply. 1. The victim with chemical spills on both arms. 2. The victim with third-degree burns of both legs. 3. The victim with first-degree burns of both hands. 4. The victim in respiratory distress. 5. The victim who inhaled smoke.

1, 2, 4, 5. Victims with chemical burns, second- and third-degree burns over more than 20% of their body surface area, and those with inhalation injuries should be transported to a burn center. The victim with first-degree burns of the hands can be treated with first aid on the scene and referred to a health care facility

The nurse is caring for an 11 month-old child in the pediatric hospital. Which of these child's findings would be a common criterion to activate the rapid response team? Select all that apply. 1. New-onset right-sided paralysis of extremities 2. Pulse rate sustained at 120/min 3. Respirations continued at 38/min 4. Sudden inability to be aroused to an awake state 5. Temperature of 101 _3 F (38.5 C)

1, 4

23. A suspected outbreak of anthrax has been transmitted by skin exposure. A client is admit- ted to the emergency department with lesions on the hands. The physician prescribes antibiotics and sends the client home. What should the nurse instruct the client to do? Select all that apply. 1. Take the prescribed antibiotics for 60 days. 2. Avoid contact with other members of the family during the treatment period. 3. Wear a mask for 60 days. 4. Expect the skin lesions to clear up within 1 to 2 weeks. 5. Wash hands frequently

1, 4 1. Take the prescribed antibiotics for 60 days. 4. Expect the skin lesions to clear up within 1 to 2 weeks. Anthrax is treated with antibiotics, and the client must continue the prescription or 60 days, even i symptoms do not persist. The client may have skin lesions at the point o contact, with macula or papule ormation; the eschar will all o in 1 to 2 weeks. Clients with anthrax are not conta- gious; the client does not need to ollow isolation procedures at home. Anthrax rom skin exposure is not transmitted by respiratory contact, and the client does not need to wear a mask.

The nurse is caring for a newly admitted client with worsening cerebral edema from increased intracranial pressure (ICP). The client is intubated and is on mechanical ventilation. Which of the following nursing interventions may help reduce ICP? Select all that apply. 1. Hyperventilate before and after suctioning 2. Maintain a quiet environment 3. Maintain neutral midline head positioning 4. Perform as many nursing interventions as possible together 5. Suction for 30 seconds to remove endotracheal tube secretions at regular intervals

1,2,3

The nurse is preparing to defibrillate a client who suddenly went into ventricular fibrillation. Which steps are essential prior to delivering a shock? Select all that apply. 1. Apply defibrillator pads 2. Call out and look around to ensure that everyone is "all clear" 3. Continue chest compressions until ready to deliver shock 4. Ensure adequate IV sedation has been given 5. Ensure that the synchronization button is turned on

1,2,3

A client with acute exacerbation of chronic obstructive pulmonary disease is intubated for mechanical ventilation. Which intervention is important in the prevention of ventilator-associated pneumonia? Select all that apply. 1. 30-45 degree elevation of the head of the bed 2. Avoid gastric over-distension 3. Maintain an endotracheal cuff pressure of at least 20 cm H2O (15 mm Hg) 4. Perform in-line endotracheal suctioning every hour 5. Perform oral care with chlorhexidine

1,2,3,5

A client in cardiac arrest has pulseless electrical activity (PEA) and is not responding to resuscitation or medication. What should the nurse consider to help identify known, treatable causes of PEA? Select all that apply. 1. Arterial blood gas shows pH 6.9 2. Breath sounds are present on only one side 3. Capillary glucose is 310 mg/dL (17.2 mmol/L) 4. Muffled heart sounds with hypotension 5. Temperature is 102.2 F (39 C)

1,2,4

Emergency medical service personnel are transporting a near-drowning victim who is currently hypothermic. Based on anticipated vital signs, the nurse needs to prepare for which interventions? Select all that apply. 1. Covering client with warm blankets 2. Logrolling the client from side to side frequently 3. Mechanical ventilation 4. Warmed blood administration 5. Warmed IV fluids

1,3,5

The emergency department nurse is assessing a client who was involved in a motor vehicle accident during which the client's head was hit. The client reports neck pain. Which actions are essential for the nurse to perform at this time? Select all that apply. 1. Apply a hard cervical collar 2. Assess neck range of motion 3. Determine if client has rectal tone 4. Position client on firm surface 5. Use logrolling if moving the client

1,4,5

While caring for a postoperative client with an invasive arterial line, the nurse identifies a large discrepancy between the arterial line reading and the manual cuff pressure. Arterial line reading: 100.62 mm Hg; manual cuff reading: 120.76 mm Hg. What interventions should the nurse take to facilitate accurate functioning of the arterial line? Select all that apply. 1. Perform a square wave test on the monitor 2. Position the client flat for all blood pressure (BP) readings 3. Recheck and compare with an automatic BP machine 4. Verify that the zero reference stopcock is leveled with the client's phlebostatic axis 5. Zero balance the system

1,4,5

The nurse is caring for client with sepsis and acute respiratory failure, who was intubated and prescribed mechanical ventilation 3 days ago. The nurse assesses for which adverse effect associated with the administration of positive pressure ventilation (PPV)? 1. Dehydration 2. Hypokalemia 3. Hypotension 4. Increased cardiac output

3

4

16. The nurse is teaching a class on bioterrorism. Which statement is the scientific rationale for designating a specific area for decontamination? 1. Showers and privacy can be provided to the client in this area. 2. This area isolates the clients who have been exposed to the agent. 3. It provides a centralized area for stocking the needed supplies. 4. It prevents secondary contamination to the health-care providers.

3. A client is brought to the emergency department with abdominal trauma following an automobile accident. The vital signs are as follows: HR 132, RR 28, BP 84/58, temp 97.0°F (36.1°C), and oxygen saturation 89% on room air. Which of the following prescriptions from the health care provider should the nurse implement first? 1. Administer 1 L 0.9% normal saline IV. 2. Draw a complete blood count (CBC) with hematocrit and hemoglobin. 3. Obtain an abdominal x-ray. 4. Insert an indwelling urinary catheter.

1. Administer 1 L 0.9% normal saline IV. The client is demonstrating vital signs consistent with fluid volume deficit, likely due to bleeding and/or hypovolemic shock as a result of the automobile accident. The client will need intravenous fluid volume replacement using an isotonic fluid (eg, 0.9% normal saline) to expand or replace blood volume and normalize vital signs. The other prescriptions can be implemented once the intravenous fluids have been initiated.

21. Thirty-two children are brought to the emergency department after a school bus accident. Two children were killed along with the three people in the car that caused the crash. Before the victims arrive, in addition to ensuring that the hospital staff are prepared for the emergency, which step should the nurse anticipate carrying out? 1. Calling the nearest crisis response team. 2. Alerting the news media. 3. Notifying the hospital volunteer office. 4. Calling the school to inform teachers of the accident.

1. Calling the nearest crisis response team. The children and their amilies are at risk or experiencing a crisis. Disaster teams are avail- able or crisis intervention in such emergencies. Usually the news media monitors emergency radio requencies and most likely are aware o the acci- dent already. Although volunteers may help in some ways, they are not responsible or crisis interven- tion. Calling the school might be done, but the emer- gency issues take precedence

25. Several clients who work in the same building are brought to the emergency department. They all have fever, headache, a rash over the entire body, and abdominal pain with vomiting and diarrhea. Upon initial assessment, the nurse finds that each client has low blood pressure and has developed petechiae in the area where the blood pressure cuff was inflated. Which isolation precautions should the nurse initiate? 1. Contact isolation with double-gloving and shoe covers. 2. Respiratory isolation with positive pressure rooms. 3. Enteric precautions. 4. Reverse isolation.

1. Contact isolation with double-gloving and shoe covers. The nurse should institute treatment or hemorrhagic ever viruses, including contact isola- tion with double-gloving and shoe covers, strict hand hygiene, and protective eyewear. The nurse should start respiratory isolation with negative pres- sure rooms, not positive pressure rooms. Enteric precautions are not needed because the virus is spread by droplet and contact. Reverse isolation protects the client; in this situation, the health care team also needs protection.

32. The nurse in the emergency department is administering a prescription for 20 mg intravenous furosemide (Lasix) which is to be given immediately. The nurse scans the client's identification band and the medication barcode. The medication administration system does not verify that furosemide is prescribed for this client; however, the furosemide is prepared in the accurate unit dose for intravenous infusion. The nurse should do which of the following next? 1. Contact the pharmacist immediately to check the prescription and the barcode label for accuracy. 2. Administer the medication now, knowing the medication is labeled and the client is identified. 3. Report the problem to the information tech- nology team to have the barcode system recalibrated. 4. Ask another nurse to verify the medication and the client so the medication can be given now

1. Contact the pharmacist immediately to check the prescription and the barcode label for accuracy. The nurse should contact the pharmacist rst to be sure the medication is labeled or admin- istration to this client. The nurse should not admin- ister the drug until all saety precautions have been observed; the nurse should also not ask another nurse to veriy the medication or client. Later, i the problem cannot be resolved with relabeling the medication, the nurse or pharmacist can contact the inormation technology team to check the barcode system.

There has been an explosion at a local chemical plant. A private car arrives at the emergency department with 4 victims whose clothes are saturated with a strong-smelling liquid. The victims are wheezing. The nurse should implement which intervention first? 1. Assessing the clients' respiratory systems 2. Decontaminating the clients 3. Donning personal protective equipment 4. Providing oxygen by nasal cannula

3

3

17. The triage nurse in a large trauma center has been notified of an explosion in a major chemical manufacturing plant. Which action should the nurse implement first when the clients arrive at the emergency department? 1. Triage the clients and send them to the appropriate areas. 2. Thoroughly wash the clients with soap and water and then rinse. 3. Remove the clients' clothing and have them shower. 4. Assume the clients have been decontaminated at the plant.

4

18. The nurse is teaching a class on biological warfare. Which information should the nurse include in the presentation? 1. Contaminated water is the only source of transmission of biological agents. 2. Vaccines are available and being prepared to counteract biological agents. 3. Biological weapons are less of a threat than chemical agents. 4. Biological weapons are easily obtained and result in significant mortality.

2

19. Which signs/symptoms should the nurse assess in the client who has been exposed to the anthrax bacillus via the skin? 1. A scabby, clear fluid-filled vesicle. 2. Edema, pruritus, and a 2-mm ulcerated vesicle. 3. Irregular brownish-pink spots around the hairline. 4. Tiny purple spots flush with the surface of the skin.

11. A client is brought to the emergency room via ambulance accompanied by her sister. The sister states, "She was playing cards with us and had a seizure. Then she had another seizure just as the first one was stopping, so I called the ambulance." The client is currently not demonstrating any seizure activity, her eyes are closed, and she does not respond to commands. Which intervention should the nurse implement first? 1. Make sure suction equipment is set up bedside. 2. Draw blood for a phenytoin (Dilantin) level. 3. Assess the client's vital signs. 4. Prepare the client for a head computed tomography (CT).

1. Following a seizure (postictal stage), the client will most likely be tired and want to sleep. Maintaining the airway is the priority; the nurse should verify that suction equipment is available in case the client aspirates or chokes. Assessing vital signs and obtaining a Dilantin level are both appropriate actions by the nurse, but assuring safety is the first priority. There is no indication of a need to obtain a head CT at this time.

4. A middle-aged man collapses in the emergency department waiting room. The triage nurse should first: 1. Gently shake the victim and ask him to state his name. 2. Perform the chin-tilt to open the victim's airway. 3. Feel for any air movement from the victim's nose or mouth. 4. Watch the victim's chest for respirations.

1. Gently shake the victim and ask him to state his name. Calling the victim's name and gently shaking the victim is used to establish unresponsiveness. The head-tilt, chin-lift maneuver is used to open the victim's airway. Feeling for any air movement from the victim's nose or mouth indicates whether the victim is breathing on his own. The rescuer can watch the victim's chest for respirations to see if the victim is breathing.

Nursing students taking a class on public health and safety are reviewing the progression of symptoms in the event of a smallpox attack. Place in order the sequence of signs and symptoms that would occur in an infected client. 1. Rash appears on the tongue and in the mouth and develops on the face and spreads to the arms, legs, and feet. 2.Rash becomes raised papules that fill with fluid, with a depressed central area resembling a navel. 3.Pustules begin to crust and scab, with shedding over a 3-week period, leaving pitted scars. 4. Papules become pustules that are round and firm, as though embedded with pellets. 5.Symptoms develop, such as high fever, headache, body aches, and malaise.

1. Symptoms develop, such as high fever, headache, body aches, and malaise. 2. Rash appears on the tongue and in the mouth and develops on the face and spreads to the arms, legs, and feet. 3. Rash becomes raised papules that fill with fluid, with a depressed central area resembling a navel. 4. Papules become pustules that are round and firm, as though embedded with pellets. 5. Pustules begin to crust and scab, with shedding over a 3-week period, leaving pitted scars.

A client is admitted to the intensive care unit with diabetic ketoacidosis. The client is most likely to exhibit which of the following arterial blood gas results? 1. pH 7.26, PaCO2 56 mm Hg (7.5 kPa), HCO3 23 mEq/L (23 mmol/L) 2. pH 7.30, PaCO2 30 mm Hg (4.0 kPa), HCO3 15 mEq/L (15 mmol/L) 3. pH 7.40, PaCO2 40 mm Hg (5.3 kPa), HCO3 24 mEq/L (24 mmol/L) 4. pH 7.58, PaCO2 48 mm Hg (6.4 kPa), HCO3 44 mEq/L (44 mmol/L)

2

A client with a bowel obstruction has been treated with gastric suctioning for 4 days. The nurse notices an increase in nasogastric drainage. Which acid-base imbalance does the nurse correctly identify? Click the exhibit button for more information. Exhibit pH 7.50 PaCO2 45 mm Hg (5.98 kPa) PaO: 90 mm Hg (12 kPa) HCO3 32 mEq/L (32 mmol/L) 1. Metabolic alkalosis, compensated 2. Metabolic alkalosis, uncompensated 3. Respiratory alkalosis; compensated 4. Respiratory alkalosis, uncompensated

2

A nurse in the intensive care unit (ICU) is caring for a client with sepsis who is on a mechanical ventilator (MV). The client is exposed to the noise of the MV, monitoring equipment, and infusion pump alarms during the day and night. What should the nurse identify as the priority nursing diagnosis (ND)? 1. Anxiety 2. Disturbed sleep pattern 3. Powerlessness 4. Risk for acute confusion

2

To obtain accurate continuous blood pressure readings via a radial arterial catheter, the nurse places the air-filled interface of the stopcock at the phlebostatic axis. Where is it located? 1. Angle of Louis at 2nd intercostal space (ICS) to left of sternal border 2. Aortic area at 2nd ICS to right of sternal border 3. Level of atria at 4th ICS, 1/2 anterior-posterior (AP) diameter 4. 5th ICS at mid clavicular line (MCL)

3

2

13. The nurse in the emergency department has admitted five (5) clients in the last two (2) hours with complaints of fever and gastrointestinal distress. Which question is most appropriate for the nurse to ask each client to determine if there is a bioterrorism threat? 1. "Do you work or live near any large power lines?" 2. "Where were you immediately before you got sick?" 3. "Can you write down everything you ate today?" 4. "What other health problems do you have?"

1

14. The health-care facility has been notified an alleged inhalation anthrax exposure has occurred at the local post office. Which category of personal protective equipment (PPE) should the response team wear? 1. Level A. 2. Level B. 3. Level C. 4. Level D.

3

15. The nurse is teaching a class on bioterrorism and is discussing personal protective equipment (PPE). Which statement is the most important fact for the nurse to share with the participants? 1. Health-care facilities should keep masks at entry doors. 2. The respondent should be trained in the proper use of PPE. 3. No single combination of PPE protects against all hazards. 4. The EPA has divided PPE into four levels of protection.

29. The nurse is triaging victims of an earthquake who were removed from a building when the earthquake occurred. Which of the following victims should be classified as red? Select all that apply. 1. A 10-year-old male with crushing chest wound, tachypnea with labored breathing, unconscious, impaled object in forehead. 2. A 49-year-old male with crushing chest pain radiating to the jaw, is diaphoretic, nauseated, and has an open fracture of the left wrist. 3. A 75-year-old female with obvious fracture of the femur, absent pedal pulses on the affected side; heart rate 110, respirations 34, skin diaphoretic; awake/alert, states pain is 10 on a scale of 1 to 10. 4. A 32-year-old female who is unconscious, 3-inch (7.6-cm) laceration to her forehead, ecchymosis behind the ears, respiratory rate 10/shallow; radial pulse is weak/thread/ rapid; no breath sounds on the right side.

2, 3 2. A 49-year-old male with crushing chest pain radiating to the jaw, is diaphoretic, nauseated, and has an open fracture of the left wrist. 3. A 75-year-old female with obvious fracture of the femur, absent pedal pulses on the affected side; heart rate 110, respirations 34, skin diaphoretic; awake/alert, states pain is 10 on a scale of 1 to 10. The client with crushing chest pain has an acute cardiac condition and can have a successul outcome i immediate interventions are initiated. The client with the open racture could be stabilized and is not a signicant actor in triage in a mass casualty incident. The client with a displaced emur racture can also be classied as immediate because the racture can impair circula- tion. There are also signs o shock and severe pain. All conditions can improve with interventions. In a mass casualty incident, the goal is to do the great- est good or the greatest number—which sometimes means that limited resources are not allocated to the very critically injured that have a very low probability o survival. The other two clients are categorized as "black"/expectant because o their critical injuries and the unavailability o advance trauma care.

17. There is a shooting in a shopping mall. Three victims with gunshot wounds are brought to the emergency department. What should the nurse do to preserve forensic evidence? Select all that apply. 1. Cut around blood stains to remove clothing. 2. Place each item of clothing in a separate paper bag. 3. Hang wet clothing to dry. 4. Refrain from documenting client statements. 5. Place bullets in a sterile container.

2, 3 2. Place each item of clothing in a separate paper bag. 3. Hang wet clothing to dry. Preserving orensic evidence is essen- tial or investigative purposes ollowing injuries that may be caused by criminal intent. The nurse should put each item o clothing in a separate paper bag and label it; wet clothing should be hung to dry. The nurse should not cut or otherwise unnecessarily handle clothing, particularly cloth- ing with such evidence as blood or body fuids. The nurse should document careully the client's description o the incident and use quotes around the client's exact words where possible. The docu- mentation will become a part o the client's record and can be subpoenaed or subsequent investiga- tion. The nurse should not handle bullets rom the client because they are an important piece o orensic evidence

26. Several clients come to the emergency depart- ment with suspected contamination by the Ebola virus. What should the nurse do? Select all that apply. 1. Call in extra staff to assist with the possibility of more clients with the same condition. 2. Isolate all the suspected clients in the emergency department in one area. 3. Call housekeeping for diluted household bleach. 4. Restrict visitors from the emergency department. 5. Quarantine all contacts.

2, 3, 4 2. Isolate all the suspected clients in the emergency department in one area. 3. Call housekeeping for diluted household bleach. 4. Restrict visitors from the emergency department. The nurse should isolate all the sus- pected clients in the emergency department in one area and restrict visitors rom the emergency depart- ment to minimize exposure to others. The nurse should also obtain diluted household bleach (1:100) to decontaminate areas suspected o coming in contact with the virus. There is no indication at this time that extra sta is needed, so the nurse should not call in extra sta, to minimize exposure to health care workers. It is not necessary to quarantine contacts until a diagnosis is conrmed. In addition, it is the role o the public health ocer to issue the quarantine i needed.

9. A client has been admitted to the emergency department diagnosed with food poisoning following an outdoor picnic. The nurse should do which of the following? Select all that apply. 1. Tell the family to discard contaminated food. 2. Collect specimens for laboratory examination. 3. Assess vital signs. 4. Initiate support for the respiratory system. 5. Monitor fluid and electrolyte status. 6. Provide antiemetics, as prescribed.

2, 3, 4, 5, 6. Food poisoning is a sudden illness that occurs after ingestion of contaminated food or drink. The nurse should first assess vital signs and then ensure that the client is not in respiratory distress, because death from respiratory paralysis can occur with botulism, fish poisoning, and other food poisonings. Measures to control nausea are important to prevent vomiting, which could exacerbate fluid and electrolyte imbalance. Because large volumes of electrolytes and water are lost by vomiting and diarrhea, fluid and electrolyte status needs to be continuously monitored. The key to treatment is determining the source and type of food poisoning. If possible, rather than discarding the food, the suspected food should be brought to the medical facility and a history obtained from the client or family.

The nurse will anticipate administration of isotonic IV fluids in which clients? Select all that apply. 1. 14-day-old client has urine output of 2 mL/kg/hr with flat fontanel 2. 3-month-old client with diarrhea has a capillary refill of 4 seconds and mottling in lower extremities 3. 8-year-old client has serum sodium of 131 mEq/L (131 mmoll) and blood urea nitrogen of 15 mg/dL (5.4 mmol/L) 4. Client is having contractions every 10 minutes and will be receiving an epidural analgesic 5. Client received a bolus of IV fluid for hyperemesis gravidarum, and urine output is 80 mL/4 hr and pulse is 120/min

2,4,5

31. A client who was a victim of a gunshot wound was treated in the emergency department and died. What should the nurse direct the unlicensed assistive personnel (UAP) to do during postmortem care? Select all that apply. 1. Remove all tubes and IV lines. 2. Cover the body with a sheet. 3. Notify the family. 4. Transport the body to the morgue. 5. Notify the chaplain.

2, 4 2. Cover the body with a sheet. 4. Transport the body to the morgue. The UAP can cover the body and trans- port it to the morgue. Deaths by gunshot wound are considered reportable deaths. All evidence in a reportable death, including tubes and IV lines, should remain intact until the coroner has been con- tacted. The health care provider should be the one to notiy the amily. The nurse should be the one to notiy the chaplain

5. A client is experiencing an allergic response. The nurse should do which of the following in order from first to last? 2. Assess the airway and breathing pattern. 3. Notify the physician. 4. Activate the rapid response team. 1. Assess for urticaria

2,1,4,3 If a client is experiencing an allergic response, the nurse's initial action is to assess the client for signs/symptoms of anaphylaxis, first checking the airway, breathing pattern and vital signs, with particular attention to signs of increasing edema and respiratory distress. The nurse should then assess for other indications of anaphylaxis, such as urticaria, feelings of impending doom or fright, weakness, sweating (because a severe systemic response to an allergen can result in massive vasodilation), increased capillary permeability, decreased perfusion, decreased venous return, and subsequent decreased cardiac output. The nurse should call the rapid response team and then notify the physician

A large-scale community disaster occurs and clients must share hospital rooms due to the rapid influx of new victims. Which room assignments are appropriate in this situation? Select all that apply. 1. 2 clients on contact isolation, one with vancomycin-resistant enterococci infection and another with methicillin-resistant Staphylococcus infection 2. 2 clients with Clostridium difficile infection, one in the stool and the other in a wound 3. A client in sickle cell disease crisis and a client with streptococcal pneumonia 4. A client who had abdominal surgery today and a client with universal precautions 5. A young client in Buck's traction with an elderly client with Parkinson's disease

2,4,5

A nurse in the emergency department is caring for a homeless client just brought in with frostbite to the fingers and toes. The client is experiencing numbness and assessment shows mottled skin. Which interventions should be included in the client's plan of care? Select all that apply. 1. Apply occlusive dressings after rewarming 2. Elevate affected extremities after rewarming 3. Massage the areas to increase circulation 4. Provide adequate analgesia 5. Provide continuous warm soaks

2,4,5

1. Three hours ago, a client was thrown From a car into a ditch, and he is now admitted to the emergency department in a stable condition with vital signs within normal limits, alert and oriented with good coloring and an open fracture of the right tibia. For which signs and symptoms should the nurse be especially alert? 1. Hemorrhage. 2. Infection. 3. Deformity. 4. Shock.

2. Because of the degree of contamination of the open fracture and the time that has passed since the accident, the risk of infection is very high. Therefore, the nurse should be especially alert for signs and symptoms of possible existing infection or early signs of infections, such as debris in the wound site, temperature abnormalities, results of laboratory studies (such as complete blood cell count and wound culture and sensitivities), or heat or redness around or in the wound. Because the client's vital signs and cardiovascular status are stable at this time, hemorrhage is not the primary concern. The client is talking coherently at this point, so his mentation does not suggest that he is in shock. However, assessment for signs and symptoms of hemorrhage and shock would certainly be ongoing. The fracture would be corrected by surgery as soon as possible, thereby minimizing the risk of deformity.

2. A client is admitted to the emergency department with a full-thickness burn to the right arm. Upon assessment, the arm is edematous, fingers are mottled, and radial pulse is now absent. The client states that the pain is 8 on a scale of 1 to 10. The nurse should: 1. Administer morphine sulfate IV push for the severe pain. 2. Call the physician to report the loss of the radial pulse. 3. Continue to assess the arm every hour for any additional changes. 4. Instruct the client to exercise his fingers and wrist.

2. Call the physician to report the loss of the radial pulse. Circulation can be impaired by circumferential burns and edema, causing compartment syndrome. Early recognition and treatment of impaired blood supply is key. The physician should be informed since an escharotomy (incision through full-thickness eschar) is frequently performed to restore circulation. Pain management is important for burn clients, but restoration of circulation is the priority. Assessments should be performed more frequently. Exercise will not restore the obstructed circulation.

A client with diabetes and an infected heel ulcer is transferred to the intensive care unit because of deteriorating condition. Based on the admission assessment, what does the nurse identify as the most likely condition'? Click the exhibit button for additional information Exhibit: Vital signs at 1000 Temperature 102.8 F (39.3 C) Blood pressure 80/60 mm Hg Heart rate 110/min Respirations 36/min. labored Sp02 89% Admission notes 1000 Client is lethargic and difficult to arouse. Large amount of purulent drainage noted from left heel ulcer. Wound culture positive for Staphylococcus aureus and serum lactate level elevated. Second fluid challenge, 0.9% normal saline solution infusing at 1000 mL/hr. CVP 2 mm Hg, PAWP 3 mm Hg. Will continue to monitor. _________________________, RN 1. Multiple organ dysfunction syndrome (MODS) 2. Sepsis 3. Septic shock 4. Systemic inflammatory response syndrome (SIRS)

3

19. A car accident involves four vehicles on a remote highway. The nearest emergency department is 15 minutes away. Which victim should be trans- ported by helicopter to the nearest hospital? 1. A 10-year-old with a simple fracture of the femur who is crying and cannot find his parents. 2. Middle-aged woman with cold, clammy skin and a heart rate of 120 bpm who is unconscious. 3. Middle-aged man with severe asthma and a heart rate of 120 bpm who is having difficulty breathing. 4. A 70-year-old man with a severe headache who is conscious.

2. Middle-aged woman with cold, clammy skin and a heart rate of 120 bpm who is unconscious. The middle-aged woman is likely in shock. She is classied as a triage level I, requiring immediate care. The child with moderate trauma is classied as triage level III (urgent and should be treated within 30 minutes). The man with asthma and the man with the severe headache are classied as triage level II (emergent) and can be transported by ambulance and reach the hospital within 15 minutes.

An unrestrained, intoxicated client accidentally drives into a steel post by the hospital emergency department door. The client's head hits the windshield, splintering the glass, and the client loses consciousness. What action is a priority for the emergency department nurse? 1. Assess client for a carotid pulse 2. Open client's airway with a head tilt, chin lift maneuver 3. Place hard cervical collar on client 4. Remove client from car onto a backboard

3

16. An apartment fire spreads to seven apartment units. Victims suffer burns, minor injuries, and broken bones from jumping from windows. Which client should be transported first? 1. A woman who is 5 months pregnant with no apparent injuries. 2. A middle-aged man with no injuries who has rapid respirations and coughs. 3. A 10-year-old with a simple fracture of the humerus who is in severe pain. 4. A 20-year-old with first-degree burns on her hands and forearms.

2. The man with respiratory distress and coughing should be transported first because he is probably experiencing smoke inhalation. The pregnant woman is not in imminent danger or likely to have a precipitous delivery. The 10-year-old is not at risk for infection and could be treated in an outpatient facility. First-degree burns are considered less urgent.

The nurse in the intensive care unit is caring for a client who is postoperative from a cardiac surgery. The client has a mediastinal chest tube. During assessment, the nurse notes bubbling in the suction control chamber. Which nursing action is appropriate'? 1. Assess the insertion site for presence of subcutaneous emphysema 2. Notify the surgeon of a large air leak 3. Take no action as the chest tube is functioning appropriately 4. Turn down the wall suction until the bubbling disappears

3

The nurse is caring for a client with a pulmonary contusion. Assessment reveals restlessness, chest pain on inspiration, diminished breath sounds, and oxygen saturation of 86%. Which acid-base imbalance does the nurse correctly identify? Click on the exhibit button for more information. Exhibit: Laboratory results pH 7.31 Pa02 76 mm Hg (10.11 kPa) PaCO2 54 mm Hg (7.18 kPa) HCO3 24 mEq/L (24 mmol/L) 1. Metabolic acidosis 2. Metabolic alkalosis 3. Respiratory acidosis 4. Respiratory alkalosis

3

The nurse is caring for a client with surgical complications who requires continuous total parenteral nutrition (TPN). The nurse assists the health care provider with the insertion of a subclavian triple lumen central venous access device. What is the nurse's priority action before initiating the TPN infusion? 1. Attach a filter to the IV tubing 2. Check baseline fingerstick glucose levels 3. Check the results of the portable chest x-ray 4. Program the electronic infusion pump

3

1

20. The client has expired secondary to smallpox. Which information about funeral arrangements is most important for the nurse to provide to the client's family? 1. The client should be cremated. 2. Suggest an open casket funeral. 3. Bury the client within 24 hours. 4. Notify the public health department.

2

21. A chemical exposure has just occurred at an airport. An off-duty nurse, knowledgeable about biochemical agents, is giving directions to the travelers. Which direction should the nurse provide to the travelers? 1. Hold their breath as much as possible. 2. Stand up to avoid heavy exposure. 3. Lie down to stay under the exposure. 4. Attempt to breathe through their clothing.

1,2,3,4

23. Which cultural issues should the nurse consider when caring for clients during a bioterrorism attack? Select all that apply. 1. Language difficulties. 2. Religious practices. 3. Prayer times for the people. 4. Rituals for handling the dead. 5. Keeping the family in the designated area.

A client is brought to the emergency department after his face slammed into a brick wall during a gang fight. Which client assessment finding is most important for the nurse to consider before inserting a nasogastric tube? 1 An ecchymotic area on the forehead 2. Frontal headache rated as 10 on a 1-10 scale 3. Nasal drainage on gauze has a red spot surrounded by serous fluid 4. Small amount of bright red blood oozing from cheek laceration

3

A client undergoing endotracheal intubation received IV sedation and succinylcholine. Shortly after respiratory status has been stabilized, the client becomes flushed, profusely diaphoretic, and has a rigid jaw. Which medication should the nurse prepare to administer? Click the exhibit button for more information. Exhibit: Vital signs Temperature 105 F (40.6 C) Blood pressure 140/90 mm Hg Heart rate 150/min Respirations 28/min O2 saturation 98% 1. IM epinephrine 2. IV atropine 3. IV dantrolene 4. IV glucagon

3

22. The nurse in the emergency department is triaging the following victims of an airplane crash. Prioritize the clients in the order in which they should be treated. 1. A 75-year-old with a 2-inch (5.1-cm) laceration to the left forearm. 2. A 22-year-old with a 2-inch (5.1-cm) laceration to the left temple, slightly confused. 3. A 14-year-old with a 2-inch (5.1-cm) laceration to chin, history of asthma, respirations 26, audible wheezing. 4. A 22-year-old female, 36 weeks pregnant with contractions every 10 to 15 minutes.

3, 2, 4, 1 3. A 14-year-old with a 2-inch (5.1-cm) laceration to chin, history of asthma, respirations 26, audible wheezing 2. A 22-year-old with a 2-inch (5.1-cm) laceration to the left temple, slightly confused. 4. A 22-year-old female, 36 weeks pregnant with contractions every 10 to 15 minutes. 1. A 75-year-old with a 2-inch (5.1-cm) laceration to the left forearm. The 14-year-old with asthma needs immediate, lie- saving interventions or the wheezing and should be seen rst. The 22-year-old who is conused should be seen next to assess or head injury; the location o the laceration could indicate a signicant blunt orce traumatic injury. The pregnant emale requires assessment but is not urgent unless other symptoms appear. The 75-year-old is nonurgent and can wait saely or several hours.

An elderly client with acute diverticulitis develops severe sepsis. The nurse is most likely to assess which manifestations of the systemic inflammatory response syndrome (SIRS) associated with sepsis? Select all that apply. 1. Central venous pressure (CVP) 18 mm Hg 2. Mean arterial blood pressure (MAP) 80 mm Hg 3. Respirations 28/min 4. Sinus tachycardia 118/min 5. Temperature 101.2 F (38.4 C) 6. White blood cell count (WBC) 13.000 ɥL with 20% bands

3,4,5,6

A child is brought to the emergency department after falling and hitting the head while playing. The child was observed in the emergency department for 3 hours and no abnormal findings were noted. Which would be reasons to advocate for the child's admission to the hospital for continued observation? Select all that apply. 1. Child has a history of muscular dystrophy 2. Child reports a constant headache rated "3" out of 10 3. Nurse smells alcohol on the parent's breath 4. Parent does not speak English 5. Parent says the child was unconscious for 5 minutes

3,5

A client in a one-car rollover presents with multiple injuries. Prioritize the interventions that must be initiated for this patient. 1. Assess for spontaneous respirations. 2. Give supplemental oxygen per mask. 3. Insert a Foley catheter if not contraindicated. 4. Obtain a full set of vital signs. 5. Remove patient's clothing. 6. Secure/start two large-bore IVs with normal saline. 7. Use the chin lift or jaw thrust method to open the airway. A 1, 7, 2, 6, 4, 5, 3 B 7, 1, 4, 2, 3, 5, 6 C 4, 1, 5, 7, 6, 3, 2 D 5, 4, 1, 7, 2, 6, 3

A 1, 7, 2, 6, 4, 5, 3

A 75-year-old client is hospitalized with chronic obstructive pulmonary disease (COPD) exacerbation. The health care provider (HCP) initiates noninvasive positive airway pressure ventilation (NIPPV) with a bilevel positive airway pressure (BIPAP) device. Prescribed medications are shown in the exhibit. Which parameter is most important for the nurse to monitor frequently in this client? Click on the exhibit button for additional information. Exhibit Medication prescription Albuterol and ipratropium: nebulizer, every 4 hours as needed Levofloxacin: 750 mg IV once daily Methylprednisolone: 40 mg IV, every 8 hours Enoxaparin: 40 mg subcutaneously, once daily 1. Blood glucose level 2. Capillary refill time 3. Extremity swelling 4. Mental status

4

The charge nurse is evaluating the skills of a new registered nurse (RN) assigned to care for a client with shock. Which action taken by the new RN indicates a need for further education? 1. Administers furosemide to a client with pulmonary artery wedge pressure (PAWP) of 24 mm Hg with cardiogenic shock 2. Increases norepinephrine infusion rate to maintain mean arterial pressure (MAP) ≥65 mm Hg in a client with anaphylactic shock 3. Moves pulse oximeter sensor from the finger to the forehead of a client with septic shock 4. Places the head of the bed (HOB) for a client with hypovolemic shock in high Fowler's position.

4

The emergency department nurse receives a client with extensive injuries to the head and upper back. The nurse will perform what action to allow the best visualization of the airway? 1. Head-tilt chin-lift in the supine position on a backboard 2. Head-tilt chin-lift in the Trendelenburg position 3. Jaw-thrust maneuver in semi-Fowler's position 4. Jaw-thrust maneuver in the supine position on a backboard

4

The flight nurse assesses an alert and oriented client at an industrial accident scene who was impaled in the abdomen by a pair of scissors. Which nursing action is the immediate priority on arrival at the scene? 1. Insert a large-bore V line and infuse normal saline 2. Obtain blood for type and crossmatch and hemoglobin 3. Remove constrictive clothing to enhance circulation 4. Stabilize the scissors with sterile bulky dressings

4

The nurse assesses diminished lung sounds and high-pitched wheezing in a client with acute asthma exacerbation. Arterial blood gas (ABG) findings are shown in the exhibit. Which acid-base imbalance does the nurse correctly identify? Click the exhibit button for more information. Exhibit pH 7.49 PaCO2 30 mm Hg (4 kPa) Pa02 79 mm Hg (10.5 kPa) HCO- 25 mEq/L (25 mmol/L) 1. Metabolic acidosis 2. Metabolic alkalosis 3. Respiratory acidosis 4. Respiratory alkalosis

4

The nurse is admitting a client with a possible diagnosis of Guillain-Barre syndrome. When collecting data to develop a plan of care for the client, the nurse should give priority to which of the following items? 1. Orthostatic blood pressure changes 2. Presence or absence of knee reflexes 3. Pupil size and reaction to light 4. Rate and depth of respirations

4

There has been a major disaster with the collapse of a large building. Hundreds of victims are expected. The emergency department nurse is sent to triage victims. Which client should the nurse tag "red" and send to the hospital first? 1. Client at 8 weeks gestation with spotting; pulse of 90/min 2. Client with bone piercing skin on leg with oozing laceration; pulse of 88/min 3. Client with fixed and dilated pupils and no spontaneous respirations 4. Client with see-saw chest movement with respirations

4

14. Thirty people are injured in a train derailment. Which client should be transported to the hospital first? 1. A 20-year-old who is unresponsive and has a high injury to his spinal cord. 2. An 80-year-old who has a compound fracture of the arm. 3. A 10-year-old with a laceration on his leg. 4. A 25-year-old with a sucking chest wound.

4 During a disaster, the nurse must make difficult decisions about which persons to treat first. The guidelines for triage offer general priorities for immediate, delayed, minimal, and expectant care. The client with a sucking chest wound needs immediate attention and will likely survive. The 80-year-old is classified as delayed; emergency response personnel can immobilize the fracture and cover the wound. The 10-year-old has minimal injuries and can wait to be treated. The client with a spinal cord injury is not likely to survive and should not be among the first to be transported to the health care facility.

Which would be the appropriate client criteria for activating a rapid response team at the hospital? Select all that apply. 1. Glasgow coma scale (GCS) score of 9 throughout shift 2. Heart rate remaining at 58 beats/min for more than 1 hour 3. Postoperative pain rated at 10 4. Respiratory rate maintaining an increase to 30 breaths/min 5. Sustained change in level of consciousness for 10 minutes

4,5

You are caring for a client with a frostbite on the feet. Place the following interventions in the correct order. 1. Immerse the feet in warm water 100° F to 105° F (40.6º C to 46.1° C). 2. Remove the victim from the cold environment. 3. Monitor for signs of compartment syndrome. 4. Apply a loose, sterile, bulky dressing. 5. Administer a pain medication. A 5, 2, 1, 3, 4 B 2, 5, 1, 4, 3 C 2, 1, 5, 3, 4 D 3, 2, 1, 4, 5

A 5, 2, 1, 3, 4 B 2, 5, 1, 4, 3 C 2, 1, 5, 3, 4 D 3, 2, 1, 4, 5

Which of these is not classified as a Category A biologic agent? A Staphylococcus enterotoxin B (SEB). B Clostridium botulinum toxin (botulism). C Bacillus anthracis (anthrax). D Francisella tularensis

A Staphylococcus enterotoxin B (SEB)

The nurse has a suggestion for improving response in the next mass casualty event. Which channel does the nurse use to introduce this idea? a. The Administrative Review b. The Critical Incident Stress Briefing (CISB) c. Through the supervisor d. Through the hospital suggestion box

A (The goal of the Administrative Review is to discern what went right and what went wrong during activation and implementation of the emergency preparedness plan. In this way, changes can be made.)

A patient who experienced a near drowning accident in a local lake, but now is awake and breathing spontaneously, is admitted for observation. Which action will be most important for the nurse to take during the observation period?

Auscultate breath sounds. rationale: Since pulmonary edema is a common complication after near drowning, the nurse should assess the breath sounds frequently. The other information also will be collected by the nurse, but it is not as pertinent to the patient's admission diagnosis.

A client comes into the emergency department (ED) clutching the chest. Which core competency for ED nurses is the first one used in this situation? 1 Assessment 2 Communication 3 Priority setting 4 Technical and procedural skills

Assessment Similar to any nurse in practice, the foundation of the emergency nurse's skill base is assessment. The nurse must be able to discern normal from abnormal rapidly and accurately, and must interpret assessment findings according to acuity and age. Communication, priority setting, and technical and procedural skills are not the first competencies to be used in this situation.

7. The American Heart Association (AHA) and Canadian Heart and Stroke Foundation guidelines urge greater availability of automated external defibrillators (AEDs) and people trained to use them. AEDs are used in cardiac arrest situations for: 1. Early defibrillation in cases of atrial fibrillation. 2. Cardioversion in cases of atrial fibrillation. 3. Pacemaker placement. 4. Early defibrillation in cases of ventricular fibrillation

4. Early defibrillation in cases of ventricular fibrillation AEDs are used for early defibrillation in cases of ventricular fibrillation. The AHA and Canadian Heart and Stroke Foundation place major emphasis on early defibrillation for ventricular fibrillation and use of the AED as a tool to increase sudden cardiac arrest survival rates.

28. Eight farm workers are admitted to the emergency department after they were splashed with "a couple of chemicals" at work 30 minutes ago. They have watery/itchy eyes, slight cough, diaphoresis, constricted pupils, and are conscious and oriented. Their clothes are wet. What action should the nurse do first? 1. Apply oxygen at 3 L per nasal cannula. 2. Remove their clothing. 3. Begin decontamination shower. 4. Isolate the clients.

4. Isolate the clients. Saety o the sta and others is the rst priority. By isolating the clients, this reduces the chance o contaminating others (secondary contami- nation). Vital signs can be obtained when it is sae— ater protecting sta, patients, and visitors rom secondary contamination. Oxygen is not indicated or any o the listed symptoms. Removing cloth- ing is important to prevent urther exposure to the client, but must be done in a sae manner to prevent secondary contamination to others. The clients can remove their own clothes and place them in plas- tic bags. Ater the saety o the sta and others is addressed, AND the acility is prepared and prop- erly trained sta is ready, the clients can be given a decontamination shower. I the sta is not trained, 911 may be the most appropriate response. Finding out which chemicals were involved is important, but does not take priority over preventing secondary contamination.

24. A severe acute respiratory syndrome (SARS) epidemic is suspected in a community of 10,000 people. As clients with SARS are admitted to the hospital, what type of precautions should the nurse institute? 1. Enteric precautions. 2. Hand-washing precautions. 3. Reverse isolation. 4. Standard precautions.

4. Standard precautions. Transmission o SARS can be contained by ollowing standard (universal) precautions, which include masks, gowns, eye protection, hand wash- ing, and sae disposal o needles and sharps. The disease is spread by the respiratory, not enteric, route. Hand washing alone is not sucient to pre- vent transmission. Reverse isolation (protection o the client) is not sucient to prevent transmission.

10. A client is admitted to the emergency department after being found in a daze walking away from her burning car after an accident. She was not injured in the accident, but the other driver died. She states, "I can't handle it anymore. There's no point to it all." The crisis nurse recommends hospital admission based on the identification of which of the following concerns? 1. The client was walking around in a daze. 2. The client has a lack of knowledge of what to do next. 3. The client is having delusions and is not in touch with reality. 4. The client is expressing helplessness and hopelessness and is a risk for suicide.

4. The client is expressing helplessness and hopelessness and is a risk for suicide. The client is demonstrating helplessness and hopelessness during a crisis, as evidenced by her statement, "I can't handle it. There is no point to it." Feelings of helplessness and hopelessness are common factors associated with suicidal ideation. Therefore, the client must be hospitalized to ensure safety to herself. There is not sufficient information to know if the client has a lack of knowledge of what to do next. The client is not having delusions, which would be evidenced by statements such as "The devil set my car on fire," not just the inability to think clearly.

A client presents with the skin appearance shown. The nurse is aware that the client's clinical manifestations are consistent with: a) Anthrax b) Smallpox c) Cyanide d) Botulism

A. anthrax

Gastric lavage and administration of activated charcoal are prescribed for an unconscious patient who has been admitted to the emergency department (ED) after ingesting 30 diazepam (Valium) tablets. Which action will the nurse plan to take first?

Assist with intubation of the patient. rationale: In an unresponsive patient, intubation is done before gastric lavage and activated charcoal administration to prevent aspiration. The other actions will be implemented after intubation.

What are the duties of the nurse case manager in the emergency department (ED)? 1 Obtain and record the client health history. 2 Provide discharge teaching. 3 Coordinate the triage process with the triage nurse. 4 Arrange appropriate referrals for the client.

Arrange appropriate referrals for the client. The nurse case manager in the ED arranges for appropriate referrals and follow-up for the client. The emergency nurse obtains and records the client history. The nurse provides discharge teaching to the client and client's family. The primary healthcare provider and assistant may coordinate the triage process with the triage nurse.

A patient's family members are in the patient room when the patient has a cardiac arrest and emergency personnel start resuscitation measures. Which action is best for the nurse to take initially?

Ask the family members about whether they would prefer to remain in the patient room or wait outside the room. rationale: Although many family members and patients report benefits from family presence during resuscitation efforts, the nurse's initial action should be to determine the preference of these family members. The other actions may be appropriate, but this will depend on what is learned when assessing family preferences.

What is the foundation of an emergency nurse's skill base? 1 Assessment 2 Clinical decision-making 3 Communication 4 Multitasking

Assessment Assessment is the foundation of any practicing nurse's skill base. The nurse must be able to rapidly and accurately interpret assessment findings according to acuity and age. Clinical decision-making, communication, and multitasking are also expected from nurses but are not as vital as the ability to make quick assessments.

Michael works as a triage nurse, and four clients arrive at the emergency department at the same time. List the order in which he will assess these clients from first to last. 1. A 50-year-old female with moderate abdominal pain and occasional vomiting. 2. A 35-year-old jogger with a twisted ankle, having a pedal pulse and no deformity. 3. An ambulatory dazed 25-year-old male with a bandaged head wound. 4. An irritable infant with a fever, petechiae, and nuchal rigidity A 1, 2, 3, 4 B 2, 1, 3, 4 C 4, 3, 1, 2 D 3, 4, 2, 1

C 4, 3, 1, 2 An irritable infant with fever and petechiae should be further assessed for other meningeal signs. The patient with the head wound needs additional history and assessment for intracranial pressure. The patient with moderate abdominal pain is uncomfortable, but not unstable at this point. For the ankle injury, a medical evaluation can be delayed 24 - 48 hours if necessary.

The client comes to the emergency department covered with coagulated blood and a white powder. The client is hysterical and fears that it is anthrax. What does the nurse do first? a. Administers antibiotics b. Provides emotional support c. Takes the client to the decontamination room d. Triages the client

C (Decontamination should precede triage. Only the most basic life-sustaining interventions should be performed before or during decontamination. The coagulated blood indicates that the major active bleeding has likely subsided.)

An ER nurse is handling a 50-year-old woman complaining of dizziness and palpitations that occur from time to time. ECG confirms the diagnosis of paroxysmal supraventricular tachycardia. The client seems worried about it. Which of the following is an appropriate response of the nurse? A "You can be discharged now; this is a probable sign of anxiety." "You have to stay here for a few hours to undergo blood tests to rule out myocardial infarction." C "We'll need to keep you for further assessment; you may develop blood clots." D "The physician will prescribe you blood-thinning medications to lessen the episodes of palpitations."

C. Paroxysmal supraventricular tachycardia (PSVT) is characterized by episodes of rapid heart rate that occurs periodically and stops on its own. PSVT decreases the cardiac output and can result to a thrombus. These clots could turn into an embolus, which could eventually lead to a stroke.

A 65-year-old patient arrived at the triage area with complaints of diaphoresis, dizziness, and left-sided chest pain. This patient should be prioritized into which category? A Non-urgent. B Urgent. C Emergent. D High urgent.

C. Emergent Chest pain is considered an emergent priority, which is defined as potentially life-threatening. Option B: Clients with urgent priority need treatment within 2 hours of triage (e.g. kidney stones). Option A: Non-urgent conditions can wait for hours or even days. Option D: High urgent is not commonly used; however, in 5-tier triage systems, High urgent patients fall between emergent and urgent in terms of the time elapsing prior to treatment.

A client arrived at the emergency department after suffering multiple physical injuries including a fractured pelvis from a vehicular accident. Upon assessment, the client is incoherent, pale, and diaphoretic. With vital signs as follows: temperature of 97°F (36.11° C), blood pressure of 60/40 mm Hg, heart rate of 143 beats/minute, and a respiratory rate of 30 breaths/minute. The client is mostly suffering from which of the following shock? A Cardiogenic. B Distributive. C Hypovolemic. D Obstructive

C. Hypovolemic Hypovolemic shock occurs when the volume of the circulatory system is too depleted to allow adequate circulation to the tissues of the body. A fractured pelvis will lose about one liter of blood hence symptoms such as hypotension, tachycardia, and tachypnea will occur. Option A: Causes of cardiogenic include massive myocardial infarction or other cause of primary cardiac (pump) failure. Option B: Distributive shock results from a relative inadequate intravascular volume caused by arterial or venous vasodilation. Option D: Obstructive shock is a form of shock associated with physical obstruction of the major vessels or the heart itself.

After exposure to radiation, a patient who is triaged into the black category is one who is considered what? a) Possible survivor b) Probable survivor c) Improbable survivor d) Designation unknown

C. improbable survivor

You are caring for clients who have been exposed to a toxic nerve agent. You will need to use diazepam with these clients. Why is diazepam given when managing the effects of toxic nerve agent toxicity? a) To control hypersecretion b) To counter excess acetylcholine c) To control possible seizures d) To reactivate acetylcholinesterase

C. to control possible seizures

Which of the following terms is the tendency for a chemical to become a vapor? a) Latency b) Persistence c) Volatility d) Toxicity

C. volatility

Following a disaster, a client's condition is serious, but she is stable enough to survive if treatment is delayed 6 to 8 hours. What category of triage would the nurse place this client? a) Red b) Green c) Yellow d) Black

C. yellow

A disoriented client involved in a motor vehicle crash is admitted to the emergency department (ED). What action must the nurse take first when assessing this client? 1 Search the client's belongings for relevant information. 2 Review previous medical records of the client. 3 Check for a medical alert necklace or bracelet. 4 Obtain a complete history from the client's family.

Check for a medical alert necklace or bracelet. The nurse must first check the client for a medical alert necklace or bracelet. This helps to obtain important medical information. The nurse then searches the client's belongings for relevant information, such as contact details or medical prescriptions. It may not be possible to obtain previous medical records or a complete history from the client's family as soon as the client is admitted to the ED.

The triage nurse is assessing the acuity level of clients rushed to the emergency department. What sign or symptom reported by the client prompts the nurse to classify the client as emergent? 1 Chest pain with diaphoresis 2 Severe abdominal pain 3 Multiple soft tissue injuries 4 Strains and sprains

Chest pain with diaphoresis The client having chest pain with diaphoresis should be classified as emergent. The "emergent" category indicates that the client has a condition that poses immediate threat to life or limb. The client with severe abdominal pain and multiple soft tissue injuries are classified as "urgent" indicating that the client needs to be treated quickly but an immediate threat to life does not exist at the moment. A client with strains and sprains is classified as "nonurgent" indicating that the client could wait for several hours without a significant risk for deterioration.

You respond to a call for help from the ED waiting room. There is an elderly patient lying on the floor. List the order for the actions that you must perform. 1. Call for help and activate the code team. 2. Instruct a nursing assistant to get the emergency cart. 3. Initiate cardiopulmonary resuscitation (CPR). 4. Perform the chin lift or jaw thrust maneuver. 5. Establish unresponsiveness. A 5, 2, 4, 3, 1 B 1, 5, 2, 4, 3 C 1, 2, 5, 4, 3 D 5, 1, 4, 3, 2

D 5, 1, 4, 3, 2 Establish unresponsiveness first. (The patient may have fallen and sustained a minor injury.) If the patient is unresponsive, get help and have someone initiate the code. Performing the chin lift or jaw thrust maneuver opens the airway. The nurse is then responsible for starting CPR. CPR should not be interrupted until the patient recovers or it is determined that heroic efforts have been exhausted. A crash cart should be at the site when the code team arrives; however, basic CPR can be effectively performed until the team arrives.

During a mass casualty, which injury receives care first? a. Abdominal evisceration b. Open fracture of the left forearm c. Sprained ankle d. Sucking chest wound

D (This casualty is a red tag, or emergent, because it can be quickly resolved until further help can be given.)

During a mass casualty, staff roles are defined. If the triage officer is incapacitated, who is the best choice for replacement? a. Communications officer b. Hospital incident commander c. Medical command physician d. Triage nurse

D (When physician resources are limited, an experienced nurse may assume this role. Whoever can best meet the needs of the clients is the best choice.)

n conducting a primary survey on a trauma patient, which of the following is considered one of the priority elements of the primary survey? A Initiation of pulse oximetry. B Complete set of vital signs. C Client's allergy history. D Brief neurologic assessment.

D Brief neurologic A brief neurologic assessment to determine the level of consciousness and pupil reaction is part of the primary survey. Vital signs, client's allergy, and initiation of pulse oximetry are considered part of the secondary survey.

A client was brought to the ED due to an abdominal trauma caused by a motorcycle accident. During the assessment, the client complains of epigastric pain and back pain. Which of the following is true regarding the diagnosis of pancreatic injury? A Redness and bruising may indicate the site of the injury in blunt trauma. B The client is symptom-free during the early post-injury period. C Signs of peritoneal irritation may indicate pancreatic injury. D. all of the above

D. All of the above Blunt injury resulting from vehicular accidents could cause pancreatic injury. Redness, bruising in the flank and severe peritoneal irritation are signs of a pancreatic injury. The client is usually pain-free during the early post-injury period, hence a comprehensive assessment and monitoring should be done.

The nursing supervisor has been notified of a large industrial fire with multiple injuries started with an electric short. When notifying the nursing peers, the nurse would classify this as which type of disaster? a) Not classified as a disaster b) A natural disaster c) A human disaster, intentionally caused d) A human disaster, unintentionally caused

D. a human disaster, unintentionally caused

During a facility disaster drill, an "injured client" presents to the emergency department with complaints of dry mouth, inability to focus his vision, and double vision. A nurse notes that the client has an unsteady gait and appears to be very weak. The client states, "My arms and legs feel like they just can't move." A nurse suspects the client may be a victim of bioterrorism with: a) anthrax. b) herpes. c) Ebola. d) botulism.

D. botulism

The nursing instructor is talking with the students about botulism. The instructor tells the students that when caring for a client with botulism, what condition is most likely to cause death? a) Diplopia b) Dysphagia c) Dysarthria d) Paralysis of respiratory muscles

D. paralysis of the respiratory muscles

A client has been exposed to a vesicant and is undergoing decontamination. Which of the following most likely would be used? a) Chlorhexidine b) Alcohol c) Sodium hypochlorite d) Soap and water

D. soap and water

The local fire department in your town is suddenly flooded with calls about a low-lying, fog-like mist with an unusual odor that is hanging over some warehouses near the river. Local authorities quickly mobilize to evacuate the area and alert the hospital. What is their possible concern? a) It might indicate possible bioterrorism. b) There might be a radiation leak in one of the warehouses. c) The fog may be the result of a dirty bomb explosion. d) This may be an incident of chemical terrorism.

D. this may be an incidence of chemical terrorism

What are the responsibilities of the emergency medicine physician? 1 Communicate client needs to support staff. 2 Direct overall care in the department. 3 Troubleshoot mechanical ventilator issues. 4 Offer advice regarding specimen collection.

Direct overall care in the department. The emergency medicine physician directs the overall care in the emergency department (ED) as well as supervises resident physicians who train in the ED. The emergency nurse communicates client needs and restrictions to support staff. The respiratory therapist assists the emergency nurse to troubleshoot mechanical ventilator issues. Laboratory technicians offer advice regarding the best practice techniques for specimen collection.

What is the nurse's first step when caring for any client in an emergency? 1 Establishing a patent airway. 2 Splinting fractures. 3 Dressing wounds. 4 Evaluating the level of consciousness.

Establishing a patent airway. Issues identified in the primary survey, an organized system to rapidly identify and effectively manage immediate threats to life, are managed first when caring for a client in the emergency department (ED). The primary survey includes airway, breathing, circulation, disability, and exposure. Therefore, establishing a patent airway is the priority for the client. Splinting fractures and dressing wounds are addressed after evaluating the consciousness level of the client.

A patient arrives in the emergency department (ED) a few hours after taking "20 to 30" acetaminophen (Tylenol) tablets. Which action will the nurse plan to take?

Give N-acetylcysteine (Mucomyst). rationale: N-acetylcysteine is the recommended treatment to prevent liver damage after acetaminophen overdose. The other actions might be used for other types of poisoning, but they will not be appropriate for a patient with acetaminophen poisoning.

During the primary survey of a patient with multiple traumatic injuries, the nurse observes that the patient's right pedal pulses are absent and the leg is swollen. Which of these actions will the nurse take next?

Initiate isotonic fluid infusion through two large-bore IV lines. rationale: The assessment data indicate that the patient may have arterial trauma and hemorrhage. When a possibly life-threatening injury is found during the primary survey, the nurse should immediately start interventions before proceeding with the survey. Although a CBC is indicated, administration of IV fluids should be started first. Completion of the primary survey and further assessment should be completed after the IV fluids are initiated.

What training does an Advanced Cardiac Life Support (ACLS) certification offer? 1 Noninvasive assessment skills for airway maintenance 2 Invasive airway management skills and electrical therapies 3 Neonatal and pediatric resuscitation 4 Validation of core emergency nursing knowledge base

Invasive airway management skills and electrical therapies An ACLS certification provides training in invasive airway management skills, pharmacology, special therapies, and electrical therapies. A Basic Life Support (BLS) certification provides training in noninvasive assessment skills for airway maintenance and cardiopulmonary resuscitation. A Pediatric Advanced Life Support certification (PALS) provides training in neonatal and pediatric resuscitation. A Certified Emergency Nurse (CEN) certification validates the core emergency nursing knowledge base.

When a patient is admitted to the emergency department after a submersion injury, which assessment will the nurse obtain first?

Lung sounds rationale: The priority assessment data are how well the patient is oxygenating, so lung sounds should be assessed first. The other data also will be collected rapidly but are not as essential as the lung sounds.

A patient arrives in the emergency department after exposure to radioactive dust. Which action should the nurse take first?

Place the patient in a shower. rationale: The initial action should be to protect staff members and decrease the patient's exposure to the radioactive agent by decontamination. The other actions can be done after the decontamination is completed.

Which protective gear is most important when caring for clients with tuberculosis or other airborne pathogens? 1 Surgical cap 2 Impervious cover gown 3 Eye protection 4 Powered air-purifying respirator (PAPR)

Powered air-purifying respirator (PAPR) The nurse uses a PAPR or a specially fitted face mask when caring for a client with airborne pathogens. These clients are preferably placed in a negative pressure room if available. When there is a high risk of contamination from blood and body fluids, other safety precautions may be used. These include use of a surgical cap, an impervious cover gown, and eye protection.

Which member of the emergency care team reports a client's mechanism of injury (MOI)? 1 Triage nurse 2 Nurse practitioner 3 Health care provider 4 Prehospital care provider

Prehospital care provider The prehospital care providers report the MOI as a communication standard when handing off care to the emergency department (ED) and trauma personnel. Clients who are brought to the ED for medical care will often relate the MOI by describing the particular chain of events that caused their injuries. The triage nurse or emergency nurses communicate this information to the nurse practitioner, health care provider, and other staff members for continuity of care.

The trauma nurse is caring for a client with a Glasgow coma scale score of 6. What immediate intervention does the nurse perform? 1 Assess the client's breath sounds and respiratory effort. 2 Observe for chest wall trauma or other physical abnormality. 3 Prepare for endotracheal intubation and mechanical ventilation. 4 Use the jaw-thrust maneuver to establish a patent airway.

Prepare for endotracheal intubation and mechanical ventilation. The nurse should prepare for endotracheal intubation and mechanical ventilation because this client is at a risk for airway compromise. The nurse assesses breath sounds and respiratory effort, observes for chest wall trauma or other physical abnormality after a patent airway is established. The jaw-thrust maneuver is used to establish a patent airway in clients with spinal injury.

What is the purpose of a Level I trauma center? 1 Provide advanced life support. 2 Stabilize clients with major injuries. 3 Provide a full continuum of trauma care. 4 Transport clients to higher level trauma centers.

Provide a full continuum of trauma care. A Level I trauma center provides the full continuum of trauma care. A Level IV trauma center provides advanced life support to clients before transporting them to higher level trauma centers. Level III trauma centers stabilize clients with major injuries. These centers also transport clients to higher level trauma centers if client needs exceed the facility's resource capabilities.

After resuscitation, a patient who had a cardiac arrest is nonresponsive to commands and therapeutic hypothermia is prescribed. Which action will the nurse include in the plan of care?

Rapidly infuse cold normal saline. rationale: When therapeutic hypothermia is used postresuscitation, cold normal saline is infused to rapidly lower body temperature to 89.6° F to 93.2° F (32° C to 34° C). Since hypothermia will decrease brain activity, neurologic assessment every 30 minutes is not needed. Sedative medications are administered during therapeutic hypothermia.

What is the role of a forensic nurse examiner? 1 Recognize evidence of physical abuse. 2 Evaluate emotional behavior. 3 Evaluate mental illness. 4 Offer basic life support intervention.

Recognize evidence of physical abuse. Forensic nurse examiners recognize evidence of physical abuse and intervene on behalf of the client. They collect forensic evidence and offer counseling to victims of interpersonal violence. A mental health nurse on the psychiatric crisis nurse team evaluates emotional behavior and mental illness in the client. The emergency medical technician offers basic life support intervention to clients.

The following actions are part of the routine emergency department (ED) protocol for a patient who has been admitted with multiple bee stings to the hands. Which action should the nurse take first?

Remove the patient's rings. rationale: The patient's rings should be removed first because it might not be possible to remove them if swelling develops. The other orders also should be implemented as rapidly as possible after the nurse has removed the jewelry.

A client with stab wounds to the abdomen is rushed to a Level III trauma center. What care does the nurse provide for the client in this facility? 1 Stabilize and arrange to transfer the client to a Level I trauma center. 2 Give first aid and transfer the client to the community hospital. 3 Provide life support and transfer the client to a Level IV trauma center. 4 Provide the full gamut of health care at the trauma center.

Stabilize and arrange to transfer the client to a Level I trauma center. A Level III trauma center would be able to stabilize this client and arrange to transfer the client to a Level I trauma center. Level III trauma centers are usually found in smaller, rural hospitals and are not equipped to provide complete care for clients with traumatic injuries. A community hospital may not be able to provide further care, so there is no need to transfer the client to such a facility. The client would not be transferred to a Level IV trauma center as they are located in remote areas and do not have the capabilities to care for this client till complete recovery. Only a Level I trauma center can provide this client with the full gamut of health care.

Which statement below is INCORRECT about the yellow triage tag color in regards to a disaster situation? A. A survivor with this tag color is seen after patients with the green tag color. B. A survivor with this tag color can have treatment delayed for an hour or less. C. A survivor with this tag color has serious injuries that could eventually lead to the compromise of breathing, circulation, or mental status, especially if treatment is delayed more than an hour or so. D. A survivor with this tag color has second priority for treatment of injuries.

The answer is A. This statement is INCORRECT. It should say: A survivor with this tag color is seen after patients with the RED (not green) tag color.

The wounded victim is able to walk and obey commands. The wounded victim is assigned what tag color? A. Green B. Red C. Yellow D. Black

The answer is A: Green.

The wounded victim is unable to walk, respiratory rate is absent but when airway is repositioned breathing is noted. The wounded victim is assigned what tag color? A. Green B. Red C. Yellow D. Black

The answer is B: Red.

or the next 7 questions, use the START method for adults to help triage the wounded that have been involved in a disaster situation. Each question will give you details on what you have assessed and you will need to use those details to help you assign a color tag to the individual: The wounded victim is unable to walk, has respiratory rate of 40, capillary refill is 6 seconds, and can't follow simple commands. The wounded victim is assigned what tag color? A. Green B. Red C. Yellow D. Black

The answer is B: Red.

A catastrophic disaster has occurred 5 miles from the hospital you are working in. The hospital's disaster plan is activated and the wounded are brought to the hospital. You're helping triage the survivors. One of the wounded is able to walk around and has minor lacerations on the arms, hands, chest, and legs. You would place what color tag on this survivor? A. Red B. Yellow C. Green D. Black

The answer is C: Green tags are for patients who have MINOR injuries. If the patient can walk around they are tagged as green. Sometimes they are referred to as the "walking wounded"

How is an unconscious male client who is brought to the emergency department (ED) eventually identified? 1 The family is asked to provide details about the client. 2 The client is provided a "John Doe" identification tag. 3 Identification is made after the client regains consciousness. 4 The client is identified by the date and time of arrival to the hospital.

The client is provided a "John Doe" identification tag. Hospitals commonly use a "Jane/John Doe" identification system for clients with unknown identity and those with emergent conditions. All clients are issued an identity bracelet at the point of entry to the ED. The nurse does not wait until the family arrives for details or until the client regains consciousness. The client is not identified by the date and time of arrival to the hospital. Other appropriate identifiers include date of birth, agency identification number, home telephone number or address, and/or Social Security number.

When rewarming a patient who arrived in the emergency department (ED) with a temperature of 87° F, which assessment indicates that the nurse should discontinue the rewarming?

The core temperature is 94° F (34.4° C). rationale: A core temperature of 89.6° F to 93.2° F (32° C to 34° C) indicates that sufficient rewarming has occurred. Dysrhythmias, hypotension, and shivering may occur during rewarming and should be treated but are not an indication to stop rewarming the patient.

There has been an explosion at a local refinery. Numerous serious and life-threatening injuries have occurred. The following clients arrive from the scene by private vehicle. Which client is considered a priority for treatment? 1 Child with an open fracture of the arm 2 Man with a contusion on the head 3 Teenager with a closed fracture of the leg 4 Woman bleeding heavily

Woman bleeding heavily The woman critically injured with trauma or an active hemorrhage is prioritized as emergent. The emergent triage category implies that a condition exists that poses an immediate threat to life or limb and should be treated immediately. Although the child with an open fracture of the arm, the man with a contusion of the head, and the teenager with a closed fracture of the leg are urgent, they are not emergent and can wait for a short time.

An unresponsive 78-year-old is admitted to the emergency department (ED) during a summer heat wave. The patient's core temperature is 106.2° F (41.2° C), blood pressure (BP) 86/52, and pulse 102. The nurse initially will plan to _____________

apply wet sheets and a fan to the patient. rationale: The priority intervention is to cool the patient. Antipyretics are not effective in decreasing temperature in heat stroke, and 100% oxygen should be given, which requires a high flow rate through a non-rebreather mask. An older patient would be at risk for developing complications such as pulmonary edema if given fluids at 1000 mL/hr.


Ensembles d'études connexes

Chapter 6: Classical Conditioning

View Set

Quiz 6 Information Security Fundamentals

View Set

CH. 14: Nonadaptive Evolution and Speciation: Urban Evolution

View Set

Fundamentals Practice Assessment A

View Set

Cognitive Psychology Exam 2 Questions

View Set

0. Vistas 2, Chapter 10, d-Imperfect Tense: 3. Activity #26 Practice- ¡Inténtalo! Estructura 10.1

View Set